Tumor OITE - 2012 2013 2014

You might also like

Download as ppt, pdf, or txt
Download as ppt, pdf, or txt
You are on page 1of 254

Tumor

University of Pennsylvania Department of Orthopaedic Surgery


2012-11 (Tumor)

• Figures 11a and 11b are the radiograph and CT scan of a


22-year-old man with a painful forearm and elbow that is
worse at night but painful at all times. Pain has been
increasing during the past 5 months. He has no history of
trauma or fevers. What is the most likely diagnosis?
– 1- Osteomyelitis
– 2- Osteoid osteoma
– 3- Ewing sarcoma
– 4- Eosinophilic granuloma
– 5- Small-cell osteosarcoma
Question 11
Question 11

• Figures 11a and 11b are the radiograph and CT scan of a


22-year-old man with a painful forearm and elbow that is
worse at night but painful at all times. Pain has been
increasing during the past 5 months. He has no history of
trauma or fevers. What is the most likely diagnosis?
– 1- Osteomyelitis
– 2- Osteoid osteoma
– 3- Ewing sarcoma
– 4- Eosinophilic granuloma
– 5- Small-cell osteosarcoma
Question 11
• Osteomyelitis is infection of bone characterized by progressive inflammatory destruction
and apposition of new bone. Seen in people with recent trauma/surgery,
immunocompromised patients, illicit IVDU, poor vascular supply, peripheral neuropathy,
and diseases such a sickle cell and diabetes.
• Osteoid osteoma manifests with increasing pain that is relieved by salicylates and other
NSAIDs. On Radiograph it shows an intensely reactive bone and a radiolucent nidus. CT
scans better elucidate the difference between nidus and reactive bone. In 50% of
patients symptoms resolve on their own with NSAIDs. Bone Scan- Always hot.
• Ewing Sarcoma is a small round cell sarcoma seen in children and young adults (5-25).
Chromosomal translocation seen is (11,22). 2 nd most common bone tumor in children,
pain is often accompanied by fever and mimics infection. Large destructive looking
appearance on radiograph & a periosteal reaction may give a sunburst or onion skin
appearance.
• Eosinophilic granuloma is disease that falls under the reticuloendothelial system. It is
usually seen as a highly destructive self limiting lesion seen in younger patients. 80% of
patients are less than 20 and seen most often in the skull, ribs, clavicle, scapula, and
mandible. Known as the great mimicker as it appears like many other lesions on imaging.
• Osteosarcoma has many types the most common being high-grade intramedullary and
this occurs in children and young adults mostly around the distal femur and proximal
tibia. Present with pain, fever, swelling and look blastic/destructive on XR and often have
Codman’s triangle.
2012-20 (Tumor)

Which inheritance pattern is characteristic of


the disorder shown in Figures 20a and 20b?
1. Autosomal-dominant
2. Autosomal-recessive
3. X-linked dominant
4. X-linked recessive
5. Mitochondrial inheritance through the
maternal line
Question 20
Which inheritance pattern is characteristic of
the disorder shown in Figures 20a and 20b?
1. Autosomal-dominant
2. Autosomal-recessive
3. X-linked dominant
4. X-linked recessive
5. Mitochondrial inheritance through the
maternal line
Multiple hereditary exostosis (MHE) is an autosomal
dominant disorder characterized by the
development of multiple osteochondromas and a
defect in the EXT-1, EXT-2, or EXT-3 gene.
EXT-1 patients have higher rates of sarcomatous
change when compared to those with a defect in
EXT-2. The reported risk for malignant
transformation to chondrosarcoma with this
condition is up to 10%.
The figures show the presence of multiple
osteochondromas, and associated bony deformity
typically found in patients with this disease. MHE
is not associated with an increase risk of
enchondroma formation, and degeneration to
osteosarcoma is very rare.
2012-25 (Tumor)

Figures 25a and 25b are the gadolinium-enhanced


MRI scans of a 68-year-old woman with
intermittent midthigh pain for 2 months. She has
an anterolateral proximal thigh mass of
approximately 9 cm. Radiographs reveal no bone
lesion, but there is a soft-tissue mass. A needle
biopsy of the thigh lesion is seen in Figure 25c.
What is the most likely diagnosis?
1. Liposarcoma
2. Fibrosarcoma
3. Desmoid tumor
4. Myxoid liposarcoma
5. Pleomorphic sarcoma
Question 25
Figures 25a and 25b are the gadolinium-
enhanced MRI scans of a 68-year-old
woman with intermittent midthigh pain for
2 months. She has an anterolateral
proximal thigh mass of approximately 9
cm. Radiographs reveal no bone lesion,
but there is a soft-tissue mass. A needle
biopsy of the thigh lesion is seen in Figure
25c. What is the most likely diagnosis?
1. Liposarcoma
2. Fibrosarcoma
3. Desmoid tumor
4. Myxoid liposarcoma
5. Pleomorphic sarcoma
• Pleomorphic sarcoma

1. Liposarcoma
Signet Ring cells

2. Fibrosarcoma
Fasiculated growth, spindle shaped cells, herringbone
fashion

3. Desmoid tumor
Rare, Painless enlarging mass, IHC = ER-beta +, assoc w
FAP & trisomy 8 or 20

4. Myxoid liposarcoma
translocation 12;16. most common (50% of liposarcs).
2012-31 (Tumor)

• Figures 31a and 31b are the radiographs of a 5-year-old child with
midtibia swelling and no pain. What is the most appropriate next
step?

• 1. Bone scan
• 2. Observation
• 3. Needle biopsy
• 4. Wide excision
• 5. Referral to a musculoskeletal tumor service for definitive
treatment
Q31
Q31

• Figures 31a and 31b are the radiographs of a 5-year-old child with
midtibia swelling and no pain. What is the most appropriate next
step?

• 1. Bone scan
• 2. Observation
• 3. Needle biopsy
• 4. Wide excision
• 5. Referral to a musculoskeletal tumor service for definitive
treatment
Q31

• Osteofibrous dysplasia is a rare, benign fibro-osseous lesion that


occurs in children and adolescents almost exclusively within the
anterior cortex of the tibia. Natural progression leads to
ossification and resolution of the disease coinciding with physeal
closure. Management consists of expectant observation. It must
be distinguished from adamantinoma which is a potentially
malignant lesion also occuring in the anterior tibial cortex.
Adamantinoma presents in young adults, is painful, and has more
aggressive pathology. Adamantinoma must be treated surgically
given its metastatic potential.
2012-35 (Tumor)
An experienced orthopaedic surgeon practicing in a community hospital setting has
a 55-year-old woman with a mixed lytic and blastic proximal humeral lesion. She
has no other symptoms and no history of malignancy. What is the most appropriate
next step in managing this condition?
 
1. Open biopsy alone
2. Open biopsy and curettage of the lesion
3. Needle biopsy performed in radiology
4. Referral to a musculoskeletal tumor center
5. Fluorodeoxyglucose positron emission tomography
4. Referral to a musculoskeletal tumor center. Biopsy of soft-tissue and bone
lesions is a difficult procedure that is fraught with complications if obtained
improperly. Occasionally, improperly performed biopsies may preclude limb-
salvage, thus greatly altering the outcome of the disease process.The standard-
of-care is to refer patients with possible musculoskeletal tumors to treatment
centers capable of proper diagnosis and management of the condition. While a
community orthopaedic surgeon may know how to properly biopsy a lesion, it is
in the patient's best interest to be referred to a treatment center prior to biopsy if
that surgeon does not feel comfortable managing the patient afterwards.

Mankin et al. review 597 musculoskeletal oncology cases to determine the


complications associated with biopsies. Of the 597 patients reviewed, 282
(47.2%) had the procedure at a referring institution. Out of all the biopsies, he
found a 17.8% rate of diagnostic error, a 19.3% chance of an incorrectly
performed biopsy causing a change in management, and a 3% incidence of an
unnecessary amputation being performed.
2012-47 (Tumor)

• Figures 47a through 47c are the radiographs and CT scan


of a 22-year-old man with a minor knee injury. Findings
from a CT scan of the chest and abdomen and a bone scan
were negative with the exception of the lesion. A biopsy
specimen is shown in Figure 47d. What is the appropriate
next step in treatment?

• 1. Observation
• 2. Marginal resection
• 3. Wide resection only
• 4. Wide resection and chemotherapy
• 5. Radiation followed by wide resection
Question 47: Images
Question 47: Answer

• The correct response is 3- wide resection


only.
Question 47: Explanation
• The clinical scenario and images depict a classic parosteal
osteosarcoma. The pathology slide confirms a relatively low grade
malignancy. The treatment of these tumors, most commonly located
on the posterior distal femur, is wide resection only. They are not
amenable to chemo or radiation. They respond well to wide resection.

• RECOMMENDED READINGS:
• Pezzillo F, Maccauro G, Nizegorodcew T, Rossi B, Gosheger G.
Resection of parosteal osteosarcoma of the distal part of the femur:
an original reconstruction technique with cement and plate. Sarcoma.
2008;2008:763056. Epub 2008 Oct 20. PubMed PMID: 18949052.
• Lewis VO, Gebhardt MC, Springfield DS. Parosteal osteosarcoma of
the posterior aspect of the distal part of the femur. Oncological and
functional results following a new resection technique. J Bone Joint
Surg Am. 2000 Aug;82-A(8):1083-8. PubMed PMID: 10954096.
2012-48 (Tumor)

• A 15-year-boy has had unrelenting


nonmechanical neck pain for 1 year. Aspirin is his
only form of pain relief. The pathology seen in
Figures 48a and 48b is typical of?

• 1. osteoblastoma
• 2. chondroblastoma
• 3. osteoid osteoma
• 4. Brodie abscess
• 5. aneurysmal bone cyst
Question 48: Images
Question 48: Answer

• The correct response is choice 3- osteoid


osteoma.
Question 48: Explanation

• This clinical scenario and imaging are consistent with an osteoid


osteoma. The patient is the typical age for osteoid osteoma, and
the history of effective pain relief with NSAIDs and especially
aspirin is classic. The location is also typical. A CT scan is the study
of choice, and will show a small nidus with reactive bone
surrounding. These can be excised surgically, but radiofrequency
ablation is also a treatment method. The majority of these will
burn out within 2-3 years.
• RECOMMENDED READINGS:
• Cronen GA, Emery SE. Benign and malignant lesions of the spine.
In: Spivak JM, Connolly PJ, eds. Orthopaedic Knowledge Update:
Spine 3. Rosemont, IL: American Academy of Orthopaedic
Surgeons; 2006:351-366.
• Kan P, Schmidt MH. Osteoid osteoma and osteoblastoma of the
spine. Neurosurg Clin N Am. 2008 Jan;19(1):65-70. Review.
PubMed PMID: 18156049.
2012-57 (Tumor)

A 51-year-old man has a slowly expanding upper-


extremity mass. Examination reveals a firm 3-cm
mass in his midvolar forearm. Radiographs are
normal. You suspect a soft-tissue sarcoma. The best
imaging study would be

1. ultrasound.
2. CT scan.
3. bone scan.
4. gadolinium-enhanced MRI scan.
5. fluorodeoxyglucose positron emission tomography.
Question 57

A 51-year-old man has a slowly expanding upper-


extremity mass. Examination reveals a firm 3-cm
mass in his midvolar forearm. Radiographs are
normal. You suspect a soft-tissue sarcoma. The best
imaging study would be

1. ultrasound.
2. CT scan.
3. bone scan.
4. gadolinium-enhanced MRI scan.
5. fluorodeoxyglucose positron emission tomography.
Question 57

The first reference is a study from 1988 that


compared surgical pathology specimens with pre-op
imaging to determine accuracy of CT vs MRI.
“Evaluation of anatomic compartments and
individual muscle involvement was more accurately
accomplished with MR imaging; nine (23%) of 40
MR studies showed tumor involvement of one or
more individual muscles that appeared normal in CT
scans.”
2012-69 (Tumor)

• 69. The cause of the condition shown in Figure 69 is


associated with which genetic abnormality?
• 1. SYT-SSX gene
• 2. t(11;22) translocation
• 3. Mutation of p53
• 4. Mutation of the EXT1 or EXT2 genes
• 5. Mutation of the GNAS gene on chromosome 20
Oncology

• 69. The cause of the condition shown in Figure 69 is


associated with which genetic abnormality?
• 1. SYT-SSX gene
• 2. t(11;22) translocation
• 3. Mutation of p53
• 4. Mutation of the EXT1 or EXT2 genes
• 5. Mutation of the GNAS gene on chromosome 20

• The figure shows multiple osteochondromas (aka


exostoses) which is indicative of MHE (multiple hereditary
exostoses). It has autosomal dominant inheritance and
associated with mutations in EXT1, EXT2, or EXT3.
2012-81 (Tumor)

• Figures 81a through 81e are the radiographs,


MRI scan, and biopsy specimens of a 41-year-
old man with mild shoulder pain. A whole-
body bone scan showed an isolated area of
uptake in the right proximal humerus. What is
the next most appropriate treatment?
– 1 – Observation
– 2 – Curettage and bone grafting
– 3 – Culture-directed antibiotics
– 4 – CT scan of the chest, abdomen, and pelvis
– 5 – Preoperative chemotherapy followed by wide
resection and reconstruction
Question 81
Question 81

• Figures 81a through 81e are the radiographs, MRI


scan, and biopsy specimens of a 41-year-old man with
mild shoulder pain. A whole-body bone scan showed
an isolated area of uptake in the right proximal
humerus. What is the next most appropriate
treatment?
– 1 – Observation
– 2 – Curettage and bone grafting
– 3 – Culture-directed antibiotics
– 4 – CT scan of the chest, abdomen, and pelvis
– 5 – Preoperative chemotherapy followed by wide resection
and reconstruction
Question 81

• The biopsy in 81e shows formation of disorganized woven bone


and broad trabeculae, and 81d is a magnified image of the first
showing multiple osteoclasts that are multinucleated. These are
classic histologic findings in Paget’s disease. On the XR there is a
blastic appearing lesion in the humeral head with increased and
decreased osteodensity. This particular image is likely in the
sclerotic phase (lytic, mixed, and sclerotic) where there is cortical
thickening with sclerotic and lucent areas. Blood tests would show
an elevated alkaline phosphatase, urinary N-telopeptide, alpha-C-
telopeptide, and deoxypyridinoline, urinary hydroxyproline, with a
normal calcium. At this point the patient has focal symptomatic
pain, which can either be observed or they can be started on
calcitonin or a bisphosphonate, the latter of which has been shown
to reduce bone turnover, improve bone pain, promote healing of
osteolytic lesions, and restore normal bone histology in Paget’s
disease with replacement of woven bone by lamellar bone in RCTs
Question 101 (Tumor)

• Figures 101a through 101f are the radiograph, CT scan, MRI scan,
and bone scan of a 28-year-old woman with a newly diagnosed
breast carcinoma who underwent a bone scan for staging. She has
no hip pain but a lesion is noted. What is the best next treatment
step?

• 1. Biopsy of the right elbow


• 2. Needle biopsy of the right femoral bone lesion
• 3. Reassurance and repeat plain radiographs in 3 months
• 4. Prophylactic intramedullary rod fixation of the right femur
• 5. Radiation to the right femoral bone lesion coordinated with
systemic therapy
Question 101
Question 101

• Figures 101a through 101f are the radiograph, CT scan, MRI scan,
and bone scan of a 28-year-old woman with a newly diagnosed
breast carcinoma who underwent a bone scan for staging. She has
no hip pain but a lesion is noted. What is the best next treatment
step?

• 1. Biopsy of the right elbow


• 2. Needle biopsy of the right femoral bone lesion
• 3. Reassurance and repeat plain radiographs in 3 months
• 4. Prophylactic intramedullary rod fixation of the right femur
• 5. Radiation to the right femoral bone lesion coordinated with
systemic therapy
Question 101
• This question stem presents a 28 yo F with newly diagnosed breast cancer that has an
incidental finding of a benign bone lesion on imaging for cancer staging. The imaging is most
consistent with fibrous dysplasia.

• Fibrous dysplasia is a developmental abnormality where bone is replaced with fibrous tissue.
It is typically seen in children and young adults. Fibrous dysplasia is usually asymptomatic
but can be associated with pathologic fracture. The proximal femur is most commonly
affected by fibrous dysplasia, but any bone may be involved. It occurs in the metaphysis or
diaphysis and can be associated with a bowing deformity (shepherd’s crook deformity).

• The appearance on x-ray images is a central, geographic lesion that looks like “ground glass”
with a well-defined sclerotic rim.

• This benign lesion can be observed with serial radiographs. If symptomatic or associated with
a pathologic fracture, curettage and bone graft with or without fixation is the treatment.
Bisphosphonates have also been shown to provide pain relief in patients with polyostotoic
fibrous dysplasia.
2012-111 (Tumor)
Figures 111a through 111c are the plain radiograph, MRI scan, and biopsy
specimen of a 14-year-old girl who had right hip pain for 2 months. After a fall,
the pain increased. What is the most likely diagnosis?

1. Osteomyelitis
2. Aneurysmal bone cyst
3. Unicameral bone cyst
4. Eosinophilic granuloma
5. Telangiectactic osteosarcoma
Question 111
Figures 111a through 111c are the plain radiograph, MRI scan, and biopsy
specimen of a 14-year-old girl who had right hip pain for 2 months. After a fall,
the pain increased. What is the most likely diagnosis?

University of Pennsylvania Department of Orthopaedic Surgery


Question 111
Figures 111a through 111c are the plain radiograph, MRI scan, and biopsy
specimen of a 14-year-old girl who had right hip pain for 2 months. After a fall,
the pain increased. What is the most likely diagnosis?

1. Osteomyelitis
2. Aneurysmal bone cyst
3. Unicameral bone cyst
4. Eosinophilic granuloma
5. Telangiectactic osteosarcoma

RECOMMENDED READINGS:
•Ramírez AR, Stanton RP. Aneurysmal bone cyst in 29 children. J Pediatr
Orthop. 2002 Jul-Aug;22(4):533-9. PubMed PMID: 12131454.
•Gibbs CP Jr, Hefele MC, Peabody TD, Montag AG, Aithal V, Simon MA.
Aneurysmal bone cyst of the extremities. Factors related to local recurrence
after curettage with a high-speed burr. J Bone Joint Surg Am. 1999
Dec;81(12):1671-8. PubMed PMID: 10608377.

University of Pennsylvania Department of Orthopaedic Surgery


Question 111
Figures 111a through 111c are the plain radiograph, MRI scan, and biopsy
specimen of a 14-year-old girl who had right hip pain for 2 months. After a fall,
the pain increased. What is the most likely diagnosis?

1. Osteomyelitis
2. Aneurysmal bone cyst
3. Unicameral bone cyst
4. Eosinophilic granuloma
5. Telangiectactic osteosarcoma
EXPLANATION:
Aneurysmal bone cysts are rare skeletal tumors that occur most frequently in people <20 years old. Pathogenesis remains uncertain, but may be
vascular, traumatic, or genetic. Differential diagnosis includes a giant cell tumor, unicameral bone cyst, and telangiectatic osteosarcoma.
Treatment consists of curettage and bone grafting with or without adjuvant treatments such as sclerotherapy, cryotherapy, or radionuclide
ablation. Figures A and B show an expansile, eccentric, lucent lesion in the right ilium with evidence of a pathologic fracture. Figure B shows a
fluid-fluid level on T2 imaging confirming a diagnosis of an aneurysmal bone cyst (ABC). Figure C shows the low-power view of an aneurysmal
bone cyst with "lakes of blood" and a benign fibrocystic lining.

Incorrect Answers:
1: Osteomyelitis is unlikely to cause the cystic expansion of bone seen in these images, and is typically characterized by bony edema on T2 MRI
in its early stages, followed by chronic reactive bone formation with involucrum and sequestrum.
3: Unicameral bone cysts (UBC) are serous filled bone lesions that occur most commonly in the proximal humerus. Unlike UBCs, long bone
ABCs are *eccentric* lesions seen in the metaphysis, and can expand to a diameter greater than that of the physis.
4. Eosinophilic granuloma is typically a solitary, self-limited form of histiocytosis X found in younger patients. It’s appearance is variable,
mimicking many other lesions, and may have a ‘punched out’ appearance in the diaphysis or result in ‘vertebra plana’ of the thoracic spine. The
lack of nuclear atypia differentiates
5. Telangiectatic osteosarcoma is

University of Pennsylvania Department of Orthopaedic Surgery


2012-122 (Tumor)
• Figures 122a through 122f are the MRI scans and synovial biopsy
specimens of a sexually active 20- year old man with a swollen,
painful knee. He has no history of fevers or trauma. Knee culture
results are negative. What is the most likely diagnosis?

• 1. Synovial sarcoma

• 2. Rheumatoid arthritis

• 3. Gonococcal arthropathy

• 4. Giant-cell tumor of bone

• 5. Pigmented villonodular synovitis


• Figures 122a through 122f are the MRI scans and synovial biopsy
specimens of a sexually active 20- year old man with a swollen,
painful knee. He has no history of fevers or trauma. Knee culture
results are negative. What is the most likely diagnosis?

• 1. Synovial sarcoma

• 2. Rheumatoid arthritis

• 3. Gonococcal arthropathy

• 4. Giant-cell tumor of bone

• 5. Pigmented villonodular synovitis


• Most often involves the knee (also in hip,
ankle, elbow, etc.);
• Most have hemorrhagic, dark brown
synovial fluid; biopsy is diagnostic;
• Most characteristic feature of PVNS on MRI is
the presence of intraarticular nodular masses
of low signal intensity on T1, T2-weighted,
and proton-density-weighted images;
• - Classic cytoarchitecture consists of subsynovial
nodular proliferation of large round, polyhedral,
or spindle cells with prominent cytoplasm and
pale nuclei;
    - Phagocytic histiocyte-like cells are also
present;
    - Lipid-laden foam cells and multinucleated
giant cells are interspersed with hemosiderin-
laden cells that resemble lining cells;
    - Giant cells appear to have phagocytic role;
    - Findings include deposition of iron, foam
cells, or giant cells;
2012-124 (Tumor)
• A 76-year-old man with a long history of tobacco use had an
acute onset of thoracic back pain with progressive numbness and
weakness in his lower extremities. Examination reveals loss of
sensation below the level of the nipples with 3/5 strength in his
legs. Figures 124a and 124b show the sagittal and axial T2-
weighted MRI scans of his thoracic spine. He also has multiple
lesions in his chest, liver, and right humerus. Following a biopsy,
which treatment is most definitive?

• 1. Chemotherapy
• 2. Radiation therapy
• 3. Palliative measures
• 4. Intravenous steroids
• 5. Surgical decompression and fusion
2012-124 (Tumor)
• A 76-year-old man with a long history of tobacco use had an
acute onset of thoracic back pain with progressive numbness and
weakness in his lower extremities. Examination reveals loss of
sensation below the level of the nipples with 3/5 strength in his
legs. Figures 124a and 124b show the sagittal and axial T2-
weighted MRI scans of his thoracic spine. He also has multiple
lesions in his chest, liver, and right humerus. Following a biopsy,
which treatment is most definitive?

• 1. Chemotherapy
• 2. Radiation therapy
• 3. Palliative measures
• 4. Intravenous steroids
• 5. Surgical decompression and fusion
• Nonoperative palliative care
– indications
– life expectancy of < 6 months
• Operative neurologic decompression,
spinal stabilization, and postoperative
radiation
– indications
– metastatic lesions to spine with neurologic deficits in
patients with life expectancy of > 6 months
2012-133 (Tumor)
Q133 Imaging
Q133 Answer
Q133 Explanation

• Vignette shows chondromyxoid fibroma


• Radiograph shows characteristic lytic
eccentric metaphyseal lesion, sharply
demarcated from adjacent bone
• Pathology
– Low power: lobules of fibromyxoid tissue
– High power: stellate cells
• Treatment is curettage with bone grafting
2012-142 (Tumor)

Surgical resection without radiation or chemotherapy is


the recommended treatment for which bone sarcoma?

1. Paget
2. Ewing
3. Osteogenic
4. Chondrosarcoma
5. Malignant fibrous histiocytoma of bone
Question 142

Surgical resection without radiation or chemotherapy is


the recommended treatment for which bone sarcoma?

1. Paget
2. Ewing
3. Osteogenic
4. Chondrosarcoma
5. Malignant fibrous histiocytoma of bone
Question 142
There is no documentation that malignant cartilaginous tumors respond to
adjuvant or neoadjuvant chemotherapy and radiation as other sarcomas do.

Wrong Answers
1. Paget-Chemotherapy is impressively beneficial in patients able to tolerate
the multidrug regimen. Irradiation is a useful but palliative adjunct.
2. Ewing-There is no debate that chemotherapy is very important in treating
patients with Ewing’s sarcoma. Radiation therapy to primary Ewing’s
achieved local control in 70% to 90% of patients.
3. Osteogenic-Studies have shown event-free and overall survival benefits in
patients treated with adjuvant chemotherapy. Neoadjuvant chemotherapy is
currenlty the standard in most centers as it helps define prognostic groups
based on histologic response and may facilitate resection by reducing tumor
size.
5. MFH-75% to 90% of patients treated with neoadjuvant chemotherapy
demonstrated a good histologic response. Radiation is occasionally used
posteroperative in the presence of marginal or contaminated wide margins.
Question 142

RECOMMENDED READINGS:

Seo SW, Remotti F, Lee FYI. Chondrosarcoma of bone.


In: Schwartz HS, ed. Orthopaedic Knowledge Update:
Musculoskeletal Tumors 2. Rosemont, IL: American
Academy of Orthopaedic Surgeons; 2007:185-195.

Thacker MM. Musculoskeletal Oncology. In: Fischgrund


JS, ed. Orthopaedic Knowledge Update 9.Rosemont, IL:
American Academy of Orthopaedic Surgeons; 2008:197-
219.
2012-153 (Tumor)

• Figures 153a and 153b are the MRI scans of a 75-year-old man with a
30-year history of swelling and drainage through a thigh wound. He
reported no fewer than 3 attempts at surgical treatment. The
drainage persisted; brief periods of antibiotic use were the exception.
The appearance of the resected skin around the sinus tract is shown
in Figure 153c. In addition to surgical drainage and debridement,
what would be the most appropriate best next step in treatment?

• 1. CT scan of the chest


• 2. Serum electrophoresis
• 3. Resection of the involucrum
• 4. Use of phenol gauze into the cavity after resection
• 5. Histologic examination of the skin at the sinus tract
• Figures 153a and 153b are the MRI scans of a 75-year-old man with a
30-year history of swelling and drainage through a thigh wound. He
reported no fewer than 3 attempts at surgical treatment. The
drainage persisted; brief periods of antibiotic use were the exception.
The appearance of the resected skin around the sinus tract is shown
in Figure 153c. In addition to surgical drainage and debridement,
what would be the most appropriate best next step in treatment?

• 1. CT scan of the chest


• 2. Serum electrophoresis
• 3. Resection of the involucrum
• 4. Use of phenol gauze into the cavity after resection
• 5. Histologic examination of the skin at the sinus tract
• Marjolin’s ulcers are a squamous cell carcinoma that develop in
patients with burn scars, or chronic drainage from osteomyelitis sinus
tracts or pressure sores. The rate of malignant squamous cell
carcinoma transformation is 1%. Esther et al performed a review of 6
patients and Tank et al published a case report of 1 patient with
Marjolin's ulcers. The average latency time between initial ulcer
formation and documentation of a malignancy was 30 years. They
noted that high histologic grade of the tumor and lymph node
metastases conferred high mortality.
2012-162 (Tumor)

• Figures 162a and 162b are the radiographs of a


56-year-old man with weight-bearing pain in the
proximal thigh. A CT scan of his abdomen is
shown in Figure 162c. The next appropriate
treatment step is?

1. observation.
2. chemotherapy.
3. radiation therapy.
4. preoperative embolization.
5. prophylactic internal fixation.
Oncology

• Figures 162a and 162b are the radiographs of a


56-year-old man with weight-bearing pain in the
proximal thigh. A CT scan of his abdomen is
shown in Figure 162c. The next appropriate
treatment step is?

1. observation.
2. chemotherapy.
3. radiation therapy.
4. preoperative embolization.
5. prophylactic internal fixation.
Bony Mets in Renal Cell Carcinoma

• This patient has weight-bearing pain, metastatic lesion on


xray in proximal femur, and CT abdomen consistent with
renal mass  this question is asking the sequence of
managing long bone mets in renal cancer

• Needs prophylactic treatment, but renal cell cancers are


hypervascular (intra-operative blood loss of 1.5-3L without
embolization)

• Embolization (Answer 4) is recommended first (within 24


hours of surgery), followed by prophylactic IMN (Answer 5);
radiation and chemotherapy (Answers 2 and 3) would come
later; observation (Answer 1) would be inappropriate due to
high fracture risk
2012-175 (Tumor)

Figures 175a through 175d are the anteroposterior and lateral


radiographs, MRI scan, and biopsy specimen of a 14-year-old girl
who has had knee pain for 6 weeks. What is the most appropriate
treatment?
 
1. Chemotherapy alone
2. Wide resection alone
3. External beam radiation alone
4. Chemotherapy and wide resection
5. Prophylactic nailing and external beam radiation
175
175

Figures 175a through 175d are the anteroposterior and lateral


radiographs, MRI scan, and biopsy specimen of a 14-year-old girl
who has had knee pain for 6 weeks. What is the most appropriate
treatment?
 
1. Chemotherapy alone
2. Wide resection alone
3. External beam radiation alone
4. Chemotherapy and wide resection
5. Prophylactic nailing and external beam radiation
175 - Explanation

• The imaging and pathology is consistent with an aggressive


bone-forming tumor of the distal femur, likely
osteosarcoma.

• Periosteal, telangiectatic, and intramedullary osteosarcoma


is treated with chemo + wide resection.

• Parosteal osteosarcoma is lower grade (has well


differentiated cells) and usually in females age 30-40, but it
ca be treated by wide resection alone.
2012-190 (Tumor)

Figures 190a through 190f are the radiographs,


MRI scans, and biopsy specimen of a 56-year-
old woman with right thigh pain. What is the
most likely diagnosis?

1. Lymphoma
2. Fibrous dysplasia
3. Multiple myeloma
4. Metastatic carcinoma
5. Eosinophilic granuloma

University of Pennsylvania Department of Orthopaedic Surgery


Question 190 - Imaging

University of Pennsylvania Department of Orthopaedic Surgery


Question 190 – Preferred Response

Figures 190a through 190f are the radiographs,


MRI scans, and biopsy specimen of a 56-year-
old woman with right thigh pain. What is the
most likely diagnosis?

1. Lymphoma
2. Fibrous dysplasia
3. Multiple myeloma
4. Metastatic carcinoma
5. Eosinophilic granuloma

University of Pennsylvania Department of Orthopaedic Surgery


Question 190- Explanation
1. Lymphoma – may appear similarly to MM on Xray and histology, but generally
contains a more mixed cellularity on histology and has a variable radiographic
appearance.
2. Fibrous dysplasia - well circumscribed lesions with ground glass or hazy appearance.
On histologic exam, has irregular foci of woven bone with whorled stroma. Described as
“alphabet soup” or “Chinese letters” appearance on histology.
3. Multiple myeloma - hematogenous malignancy of monoclonal plasma cells (histology
slide). MM leads to lytic bone lesions due to osteoclastic stimulation and uncoupling of
osteoclastic and osteoblastic activity. On T1-weighted spin-echo images, a lesion appears
as a hypointense area on a background of hyperintense marrow. T2-weighted spin-echo
sequences reveal hyperintense focal images relative to the hypointense marrow.
4. Metastatic carcinoma – sheets of epithelial cells on histology. May be lytic or blastic
lesions on Xray.
5. Eosinophilic granuloma (Histiocytosis X) – lesions contain a mixture of cells on
histology, including monocytes and eosinophils. On Xray, lesions are in the center of the
medullary cavicty and may cause endosteal scalloping or periosteal reaction.

Scharschmidt TJ, Lindsey JD, Becker PS, Conrad EU. Multiple myeloma: diagnosis and orthopaedic
implications. J Am Acad Orthop Surg. 2011 Jul;19(7):410-9. Review. PubMed PMID: 21724920.

University of Pennsylvania Department of Orthopaedic Surgery


2012-216 (Tumor)
Figures 216a and 216b are the biopsy specimens of a 30-
year-old man with an enlarging gluteal mass.
What is the most likely diagnosis?

1. Fibrosarcoma
2. Desmoid tumor
3. Nodular fasciitis
4. Pleomorphic sarcoma
5. Atypical lipomatous tumor

University of Pennsylvania Department of Orthopaedic Surgery


University of Pennsylvania Department of Orthopaedic Surgery
216. Figures 216a and 216b are the biopsy specimens of a
30-year-old man with an enlarging gluteal mass.
What is the most likely diagnosis?

1. Fibrosarcoma
2. Desmoid tumor
3. Nodular fasciitis
4. Pleomorphic sarcoma
5. Atypical lipomatous tumor

University of Pennsylvania Department of Orthopaedic Surgery


2. Desmoid tumor
This question mainly relies on the ability to examine the path photos. The first thing
to note is the basic tissue type being seen. Soft tissue tumors can arise from any of
the following: synovial tissue, lipogenic, muscle, angiogenic or fibrogenic. With no
cyst like cells or fat droplets you can eliminate lipogenic or synovial lesions. With no
RBC’s or endothelium you can eliminate angiogenic. You can eliminate pleiomorphic
sarcoma in this case because this has similar appearing cells. This leaves you with
fibrogenic and muscle lesions. Distinguishing a low grade fibrosarcoma with
desmoids tumors, also known as fibromatosis, can be difficult. Desmoids are
composed of normal-appearing fibroblastic cells proliferating in an abundant stroma.
Microscopically, the proliferation consist of elongated, slender, spindle-shaped cells
of uniform appearance surround by and separated from one another by abundant
collagen with little or no cell-to-cell contact. There usually is sparse cellularity. This
is in contrast to fibrosarcoma and nodular fascitis in which there is typically more
abundant cellularity. Furthermore, desmoid tumors typically have few mitoses and
necrosis is rare. This is again in contrast to fibrosarcoma and nodular fascitis in
which higher mitotic activity is seen with usual areas of necrosis. Pleiomorphic
sarcoma is wrong

University of Pennsylvania Department of Orthopaedic Surgery


2012-222 (Tumor)

222. Ewing sarcoma is most commonly


associated with which translocation?

1. t(9;22)
2. t(2;13)
3. t(11;22)
4. t(12;16)
5. t(X;18)

University of Pennsylvania Department of Orthopaedic Surgery


222. Ewing sarcoma is most commonly
associated with which translocation?

1. t(9;22)
2. t(2;13)
3. t(11;22)
4. t(12;16)
5. t(X;18)

University of Pennsylvania Department of Orthopaedic Surgery


3. t(11;22)

Straight memorization.
Eastiest way to remember:
Patrick Ewing wore number 33
(11+22)

University of Pennsylvania Department of Orthopaedic Surgery


2012-228 (Tumor)

Which figure matches the radiographs seen in


Figures 228a and 228b from a 15-year-old boy with
a 3-month history of knee pain?
 
1. Figure 228c
2. Figure 228d
3. Figure 228e
4. Figure 228f
5. Figure 228g

University of Pennsylvania Department of Orthopaedic Surgery


University of Pennsylvania Department of Orthopaedic Surgery
University of Pennsylvania Department of Orthopaedic Surgery
Question 228

Which figure matches the radiographs seen in


Figures 228a and 228b from a 15-year-old boy with
a 3-month history of knee pain?
 
1. Figure 228c
2. Figure 228d
3. Figure 228e
4. Figure 228f
5. Figure 228g

University of Pennsylvania Department of Orthopaedic Surgery


The xrays show a permeative metaphyseal lesion with soft tissue
extension, new bone formation, periosteal reaction, and sunburst
appearance. This constellation of radiographic findings is classic
for osteosarcoma. These tumors typically occur in the first three
decades of life with a second peak beyond the sixth decade.
Histologically, osteosarcoma presents as woven bone with no
osteoblastic rimming and a malignant spindle cell stroma
(increased cellularity, spontaneous necrosis, significant atypia or
pleomorphism, high mitotic rate with many abnormal mitoses).
The correct answer is therefore (3) Figure 228e.

RECOMMENDED READINGS:
Forest M, Tomeno B, Vanel D, eds. Orthopedic Surgical Pathology:
Diagnosis of Tumors and Pseudotumoral Lesions of Bone and
Joints. New York, NY: Churchill Livingstone; 1998.
Messerschmitt PJ, Garcia RM, Abdul-Karim FW, Greenfield EM,
Getty PJ. Osteosarcoma. J Am Acad Orthop Surg. 2009
Aug;17(8):515-27. Review. PubMed PMID: 19652033.

University of Pennsylvania Department of Orthopaedic Surgery


2012-239 (Tumor)

University of Pennsylvania Department of Orthopaedic Surgery


Question 239

University of Pennsylvania Department of Orthopaedic Surgery


Question 239

Bone

Multiple eccentric nuclei


Found in bone

Answer = 2 Megakaryocyte:
Multiple central nuclei
Found in bone marrow

University of Pennsylvania Department of Orthopaedic Surgery


2012-269 (Tumor)

• A 14-year-old girl has a painless deformity of the right


tibia. A radiograph from 2 years ago is seen in Figure
269a; nothing was done at that time. Her current
radiograph is seen in Figure 269b. She has no pain,
fever, or drainage. What is the most likely diagnosis?
 
1. Adamantinoma
2. Fibrous dysplasia
3. Osteofibrous dysplasia
4. Nonossifying fibroma
5. Chronic osteomyelitis
2012-269
2012-269

• A 14-year-old girl has a painless deformity of the right


tibia. A radiograph from 2 years ago is seen in Figure
269a; nothing was done at that time. Her current
radiograph is seen in Figure 269b. She has no pain,
fever, or drainage. What is the most likely diagnosis?
 
1. Adamantinoma
2. Fibrous dysplasia
3. Osteofibrous dysplasia
4. Nonossifying fibroma
5. Chronic osteomyelitis
2012-269
• DDx of tibial lesions: osteofibrous dysplasia vs adamantinoma
• Osteomyelitis unlikely due to no pain, fevers, or drainage
• Adamantinoma: soft tissue and intramedullary extension

Lesion Osteofibrous Dysplasia Adamantinoma Fibrous Dysplasia Non-ossifying Fibroma


Radiology Soap-bubble Punched - Bubbly
appearance out lesion lytic lesion
surrounded
by sclerotic
margin

Histology Osteoblastic rimming Nests of epithelial cells


in glandular pattern

Location Tibia Tibia Proximal femur, rib, Long bone metaphysis


tibia, maxilla

Pain? Usually painless swelling Usually painful swelling Asymptomatic Usually asymptomatic

Treatment Observation Surgical resection Observation for Observation; curettage


asymptomatic, ORIF and bone graft for large
for impending fxs symptomatic lesions
2012-273 (Tumor)

• A 10-year-old boy twisted his ankle while skateboarding


and has pain and swelling around the lateral ankle just
distal to the fibula. Radiographs are obtained and a
lesion is identified in the distal tibia as seen in Figures
273a and 273b. Two weeks later he has no pain to
palpation in the region and denies antecedent pain. What
is the most appropriate treatment for this lesion?

1. Biopsy
2. Naprosyn
3. Observation
4. Radio frequency ablation
5. Curettage and bone grafting
2012-273
2012-273

• A 10-year-old boy twisted his ankle while skateboarding


and has pain and swelling around the lateral ankle just
distal to the fibula. Radiographs are obtained and a
lesion is identified in the distal tibia as seen in Figures
273a and 273b. Two weeks later he has no pain to
palpation in the region and denies antecedent pain. What
is the most appropriate treatment for this lesion?

1. Biopsy
2. Naprosyn
3. Observation
4. Radio frequency ablation
5. Curettage and bone grafting
2012-273

• This lesion is a non-ossifying fibroma


– Benign lesion, most common in childhood
– Found in metaphyseal regions (distal femur, proximal tibia, distal tibia)
– Classically discovered as incidental Xray finding after trauma
• Usually asymptomatic
• Xray: “bubbly” lytic lesion with sclerotic margin
• Treatment:
– Asymptomatic lesions: observation – most spontaneously resolve
– Large painful lesions w/wo pathologic fx: curettage and bone graft,
possible ORIF

• In general:
– Observation for benign and asymptomatic lesions
– Surgical intervention for malignant and/or painful and symptomatic
lesions
Question 4
A 60yo F has a high-grade pleomorphic undifferentiated
sarcoma of the thigh.
•Which figure supports this pathologic diagnosis?
Question 4

• A 60-year-old woman has a high-grade


pleomorphic undifferentiated sarcoma of the
thigh.
– Which figure supports this pathologic diagnosis?

1. Figure 4a
2. Figure 4b
3. Figure 4c
4. Figure 4d
5. Figure 4e
Pleomorphic Liposarcoma

• Soft-tissue tumor of the thigh that is a high-grade, unclassified/undifferentiated


pleomorphic malignant soft-tissue sarcoma (previously part of the entity known as
malignant fibrous histiocytoma). It is characterized by “sheets of epithelioid tumor
cells with a vesicular nuclear pattern without histologic evidence of lipogenic
differentiation”. Rate of metastasis is >50%.

MedScape©

• Chung L, Lau SK, Jian Z, Loera S, Bedel V, Ji J, Weiss LM, Chu PG. Overlapping features between
dedifferentiated liposarcoma and undifferentiated high-grade pleomorphic sarcoma. Am J Surg
Pathol 2009; 33:1594-1600.
Question 17

Which soft-tissue sarcoma is most likely to develop lymphatic metastasis?


•1. Liposarcoma
•2. Leiomyosarcoma
•3. Synovial sarcoma
•4. Myxoid liposarcoma
•5. Pleomorphic sarcoma
Question 17

Which soft-tissue sarcoma is most likely to develop lymphatic metastasis?


•1. Liposarcoma
•2. Leiomyosarcoma
•3. Synovial sarcoma
•4. Myxoid liposarcoma
•5. Pleomorphic sarcoma
Question 17

Sarcomas typically metastasize to the lungs. However, synovial sarcoma is


one of the rare sarcomas that can also metastasize to the lymph nodes.
Synovial sarcomas are most commonly associated with fusion protein SYT-
SSX1.

Randall RL. Malignant soft-tissue tumors. In: Schwartz, HS, ed. Orthopaedic
Knowledge Update: Musculoskeletal Tumors 2. Rosemont, IL: American
Academy of Orthopaedic Surgeons; 2007:277-287.
Riad S, Griffin AM, Liberman B, Blackstein ME, Catton CN, Kandel RA,
O’Sullivan B, White LM, Bell RS, Ferguson PC, Wunder JS. Lymph node
metastasis in soft tissue sarcoma in an extremity. Clin Orthop Relat Res 2004
Sep:(426): 129-34. PubMed PMID: 15346063.
Oncology Question 25

Figures 25a through 25c are the axial T1 and postcontrast


MRI scans and biopsy specimen of a 35-year-old man with a
painless right thigh mass. He noticed the mass about 2 weeks
ago and is unsure if it has changed in size. Which
translocations most commonly is associated with this type of
tumor?

1)t(2;13)(q35;q14)
2)t(12;22)(q13;q12)
3)t(12;16)(q13;p11)
4)t(X;18)(p11;q11)
5)t(17;22)(q22;q13)
Oncology Question 25
Oncology Question 25

Figures 25a through 25c are the axial T1 and postcontrast


MRI scans and biopsy specimen of a 35-year-old man with a
painless right thigh mass. He noticed the mass about 2 weeks
ago and is unsure if it has changed in size. Which
translocations most commonly is associated with this type of
tumor?

1)t(2;13)(q35;q14)
2)t(12;22)(q13;q12)
3)t(12;16)(q13;p11)
4)t(X;18)(p11;q11)
5)t(17;22)(q22;q13)
Oncology Question 25

• Chromosomal abnormalities can help


diagnose and clarify prognosis for MSK
tumors, t(12;16)(q13;p11) is diagnostic of
myxoid/round cell liposarcoma.
• Krishnan B, Khanna G, Clohisy D. Gene
translocations in musculoskeletal neoplasms.
Clin Orthop Relat Res. 2008;466:2131-2146.
Oncology

34

• Figures 34a through 34d are the radiographs and biopsy specimen
of a 68-year-old woman with an 8-month history of a slowly
enlarging, painful distal left thigh mass. What is the recommended
treatment?

• 1. Surgery alone
• 2. Radiotherapy alone
• 3. Radiotherapy and surgery
• 4. Neoadjuvant chemotherapy and surgery
• 5. Neoadjuvant chemotherapy, radiotherapy, and surgery
34
34

• Oncology 34. Figures 34a through 34d are the radiographs and
biopsy specimen of a 68-year-old woman with an 8-month history
of a slowly enlarging, painful distal left thigh mass. What is the
recommended treatment?

• 1. Surgery alone
• 2. Radiotherapy alone
• 3. Radiotherapy and surgery
• 4. Neoadjuvant chemotherapy and surgery
• 5. Neoadjuvant chemotherapy, radiotherapy, and surgery
34
• Chondrosarcoma is a malignant mesenchymal tumor that produces cartilage matrix. This
tumor is more common in people over 60.

• On radiographs, chondrosarcoma is a fusiform, lucent defect with scalloping of the inner


cortex and periosteal reaction. Extension into the soft tissue may be present as well as
punctate calcification of the cartilage matrix. CT is helpful in defining the integrity of the
cortex and distribution of calcification. MRI is invaluable in surgical planning as it
demonstrates intraosseus and soft tissue involvement of the tumor.

• Histologically, chondrosarcoma is differentiated from benign cartilage growths by enlarged


plump nuclei, multiple cells per lacunae, binucleated cells, and hyperchromic nuclear
pleomorphism. Low grade chondrosarcoma is very close in appearance to enchondromas and
osteochondromas. High grade chondrosarcomas have increased cellularity, atypia and
mitoses.

• Treatment of chondrosarcoma is wide surgical excision. There is a very limited role


for chemotherapy or radiation. Biopsies must be planned with future tumor excision in
mind.

• RECOMMENDED READINGS
• Seo SW, Remotti F, Lee FYI. Chondrosarcoma of bone. In: Schwartz HS, ed. Orthopaedic Knowledge Update: Musculoskeletal Tumors 2. Rosemont, IL: American
Academy of Orthopaedic Surgeons; 2007:185-195.
• Greenspan A, Jundt G, Remagen W, eds. Differential Diagnosis in Orthopaedic Oncology. 2nd ed. Philadelphia, PA: Lippincott Williams & Wilkins; 2007:158-256.
Question 47

Figures 47a through 47d are the plain radiographs, axial MRI scan, and

biopsy specimen of an 8-year- old boy with progressive right elbow pain

that awakens him from sleep. Examination reveals soft-tissue fullness

around his elbow and pain with active or passive motion. What is the

most likely diagnosis?

1. Lymphoma

2. Osteomyelitis

3. Osteogenic sarcoma

4. Ewing sarcoma

5. Langerhans cell histiocytosis


University of Pennsylvania Department of Orthopaedic Surgery
University of Pennsylvania Department of Orthopaedic Surgery
Question 47

Figures 47a through 47d are the plain radiographs, axial MRI scan,

and biopsy specimen of an 8-year- old boy with progressive right

elbow pain that awakens him from sleep. Examination reveals soft-

tissue fullness around his elbow and pain with active or passive

motion. What is the most likely diagnosis?

1. Lymphoma

2. Osteomyelitis

3. Osteogenic sarcoma

4. Ewing sarcoma

5. Langerhans cell histiocytosis


University of Pennsylvania Department of Orthopaedic Surgery
Langerhans cell histiocytosis made up of three clinical entities with similar indistinguishable pathology:
eosinophilic granuloma (osseous, pulmonary), Hand-Schuller-Christian disease (multiple organ systems,
skull base), Letterer-Siwe’s disease (severe, involves abdominal viscera)
PRESENTATION
•Eosinophilic granuloma: children <15, bone lesions usually
asymptomatic but may be site of fracture, pain, swelling,
deformity, a soft tissue component
• In children there is usually a pulmonary component
•Osseous lesion: lytic, destructive lesions, frequently of flat bone;
can have punched out appearance, may have retained bone
fragment in middle of lesion
LPF: large number of eosinophils, histiocytes
•Should evaluate with CT scan
HPF: Langerhans cell: vesicular nucleus, groove
•Lesions in long bone usually in femur/humerus/tibia
parallel to the long axis, vacuolated cytoplasm
• Usually METAPHYSEAL and can extend into
-Notice this is not Hodkins Lymphoma because
physis/epiphysis
there are not Reed-Sternberg cells
• Appearance is endosteal scalloping

Langerhans cell histiocytosis can be diagnosed via a


clinical presentation including osseous lytic lesions
that are most commonly asymptomatic but may
exhibit a soft tissue component along with the
pathology of the lesion demonstrating the
Langerhans cell in the setting of a large number of
eosinophils and histiocytes.

RECOMMENDED READINGS

Hoover KB, Rosenthal DI, Mankin H. Langerhans cell histiocytosis. Skeletal Radiol. 2007 Feb;36(2):95- 104. Epub 2006 Oct 7. Review. PubMed

PMID: 17028900.

Lichtenstein L, Jeffe HL. Eosinophilic granuloma of bone: With report of a case. Am J Pathol 1940 Sep;16(5):595-604.3. PubMed PMID: 19970524.

University of Pennsylvania Department of Orthopaedic Surgery


Question 60

• Figures 60a through 60e are the radiographs, MRI


scan, and biopsy specimen of a 17-year-old boy with a
3-month history of left hip pain that is constant and
not relieved by anti-inflammatory medication. What is
the most likely diagnosis?

1. Enchondroma
2. Ewing sarcoma
3. Osteoblastoma
4. Chondrosarcoma
5. Chondroblastoma
60a 60b 60c

60d 60e
Question 60

• Figures 60a through 60e are the radiographs, MRI


scan, and biopsy specimen of a 17-year-old boy with a
3-month history of left hip pain that is constant and
not relieved by anti-inflammatory medication. What is
the most likely diagnosis?

1. Enchondroma
2. Ewing sarcoma
3. Osteoblastoma
4. Chondrosarcoma
5. Chondroblastoma
Question 60

Chondroblastomas are epiphyseal lesions in young adults, most


commonly found in distal femur, proximal tibia, proximal humerus,
proximal femur and apophysis or triradiate cartilage of pelvis; XR
findings include well-circumscribed epiphyseal lytic lesions with thin
rim of sclerotic bone sharply demarcated from normal medullary
cavity with or without stippled calcifications; histology reveals
chondroblasts in “cobblestone” or “chickenwire” pattern with
scattered multinucleated giant cells in areas of chondroid matrix

Sailhan F, Chotel F, Parot R, SOFOP. Chondroblastoma of bone in a pediatric population.


J Bone Joint Surg Am. 2009 Sep;91(9):2159-68. PubMed PMID: 19723993.
Weber KL, O’Connor MI. Benign cartilage tumors. In: Schwartz HS, ed. Orthopaedic
Knowledge Update: Musculoskeletal Tumors 2. Rosemont, IL: American Academy of
Orthopaedic Surgeons; 2007:103-120.
Greenspan A, Jundt G, Remagen W, eds. Differential Diagnosis in Orthopaedic Oncology.
2nd ed. Philadelphia, PA: Lippincott Williams & Wilkins; 2007:158-256.
Question 71 Tumor

What is the treatment recommendation for an


American Joint Committee on Cancer stage IIB
(Enneking stage IIB) malignant fibrous histiocytoma
of bone?
1. Wide excision alone
2. Radiotherapy and wide excision
3. Chemotherapy and radiation
4. Chemotherapy and wide excision
5. Chemotherapy, wide excision, and radiotherapy
Question 71 Tumor

What is the treatment recommendation for an


American Joint Committee on Cancer stage IIB
(Enneking stage IIB) malignant fibrous histiocytoma
of bone?
1. Wide excision alone
2. Radiotherapy and wide excision
3. Chemotherapy and radiation
4. Chemotherapy and wide excision
5. Chemotherapy, wide excision, and radiotherapy
Question 71 Tumor

MFH-B (aka pleomorphic sarcoma) presents in older patients as a lytic


lesion pn plain film with little to no periosteal reaction. It can
appear similar to osteosarcoma. Its chemotherapy regimen is
similar to that of osteosarcoma. It is treated with 2-3 months of
neoadjuvant chemotherapy, wide excision, and 6-12 months of
post operative chemotherapy.

Malignant Fibrous Histiocytoma is treated with neoadjuvant


chemotherapy, wide resection, and post operative chemotherapy.

RECOMMENDED READINGS
Wittig JC, Villalobos CE. Other skeletal sarcomas. In: Schwartz HS, ed.
Orthopaedic Knowledge Update: Musculoskeletal Tumors 2.
Rosemont, IL: American Academy of Orthopaedic Surgeons;
2007:197-204.
Jeon DG, Song WS, Kong CB, Kim JR, Lee SY. MFH of bone and
osteosarcoma show similar survival and chemosensitivity. Clin
Orthop Relat Res. 2011 Feb;469(2):584-90. Epub 2010 Jun 18.
PubMed PMID: 20559764.
Onc
Question 79

Figures 79a through 79d are the plain radiographs and axial CT scans of an 80-
year-old woman with severe dementia and a newly noted thigh mass.
Examination reveals a large, nonmobile anterior thigh mass that is minimally
tender. What is the best next treatment step?

1. Biopsy

2. Chest CT scan

3. Observation

4. Wide resection

5. Marginal resection
Imaging
Question 79

Figures 79a through 79d are the plain radiographs and axial CT scans of an 80-
year-old woman with severe dementia and a newly noted thigh mass.
Examination reveals a large, nonmobile anterior thigh mass that is minimally
tender. What is the best next treatment step?

1. Biopsy

2. Chest CT scan

3. Observation

4. Wide resection

5. Marginal resection
Question 79 Explained

• Beiner JM, Jokl P. Muscle contusion injuries: current treatment options. J


Am Acad Orthop Surg. 2001 Jul-Aug;9(4):227-37. Review. PubMed PMID:
11476532.
– Myositis ossificans has long been recognized as a leading complication
of muscle contusion injury. Although certain regions are more prone to
the development of myositis, such as the quadriceps and brachialis,
the mechanisms have not been clearly established.
– The typical radiographic appearance of myositis ossificans is
circumferential calcification with a lucent center, and a radiolucent cleft
(string sign) that separates the lesion from the cortex of the adjacent
bone
– CT appearances are similar to those of plain radiography,
demonstrating mineralization proceeding from the outer margins
towards the center
– Treatment is usually nonoperative
– Myositis ossificans is a common complication of muscle contusion and
is initially treated with observation
Question 86

Figures 86a through 86c are the radiographs


and biopsy specimen of a 14-year-old boy who
has had left knee pain for 4 weeks. What is the
most likely diagnosis?

1.Chondroblastoma
2.Chondrosarcoma
3.Parosteal osteosarcoma
4.Osteoblastoma
5.Osteosarcoma
Question 86
Question 86

Figures 86a through 86c are the radiographs


and biopsy specimen of a 14-year-old boy who
has had left knee pain for 4 weeks. What is the
most likely diagnosis?

1.Chondroblastoma
2.Chondrosarcoma
3.Parosteal osteosarcoma
4.Osteoblastoma
5.Osteosarcoma
Question 86
• Explanation: Osteosarcoma is the most common bone sarcoma
characterized by the production of both osteoid and bone. Incidence
peaks in the 2nd decade of life with rapid bone turnover during
growth spurt. Found at sites of high bone turnover: distal femur,
prox tibia, prox humerus. Classic radiographic findings are
aggressive metaphyseal bone lesions with ill defined boarders and
osteoblastic/lytic lesions. Histologically, malignant cells are found
directly adjacent to osteoid (eosinophilic acellular amorphous
material).

• Osteosarcoma is commonly found in the 2nd decade of life at


areas of high bone turnover with osteoid present on histology.

• Messerschmitt PJ, Garcia RM, Abdul-Karim FW, Greenfield EM, Getty PJ. Osteosarcoma. J Am Acad
Orthop Surg. 2009 Aug;17(8):515-27. Review. PubMed PMID: 19652033.
• Hornicek FJ. Osteosarcoma of bone. In: Schwartz HS, ed. Orthopaedic Knowledge Update:
Musculoskeletal Tumors 2. Rosemont, IL: American Academy of Orthopaedic Surgeons; 2007:163-174.
• Greenspan A, Jundt G, Remagen W, eds. Differential Diagnosis in Orthopaedic Oncology. 2nd ed.
Philadelphia, PA: Lippincott Williams & Wilkins; 2007:40-157.
Question 109
• Figures 109a through 109d are the plain
radiographs and axial T2-weighted MRI scans of a
30-year-old woman who has had right hip pain
for the past month. Examination reveals an
antalgic gait and a firm, fixed proximal femoral
mass. What is the best next treatment step?
1. Biopsy
2. Observation
3. Chemotherapy
4. Internal fixation
5. Radiation therapy
Question 109
• Figures 109a through 109d are the plain
radiographs and axial T2-weighted MRI scans of a
30-year-old woman who has had right hip pain
for the past month. Examination reveals an
antalgic gait and a firm, fixed proximal femoral
mass. What is the best next treatment step?
1. Biopsy
2. Observation
3. Chemotherapy
4. Internal fixation
5. Radiation therapy
Explanation

• Biopsy is required to histopathologically


confirm a clinical and radiographic impression
of osteosarcoma, which is classically
demonstrated by an aggressive, osteolytic or
osteoblastic metaphyseal lesion with ill-
defined borders and an associated soft tissue
mass.

• RECOMMENDED READINGS:
• Gibbs CP Jr, Weber K, Scarborough MT. Malignant bone tumors. Instr Course Lect
2002:51;413-28. PubMed PMID: 12064130.
• Messerschmitt PJ, Garcia RM, Abdul-Karim FW, Greenfield EM, Getty PJ.
Osteosarcoma. J Am Acad Orthop Surg. 2009 Aug;17(8):515-27. Review. PubMed
PMID: 19652033.
Question #127 (Oncology)

Figures 127a through 127c are the femur radiographs of a 76-year-old


man who is a communityambulator. This man, who has biopsy-proven
metastatic prostate cancer, has been experiencing left hip and thigh pain
while walking for 3 weeks. What is the best next treatment step?

1. Total hip arthroplasty


2. Long cephalomedullary fixation
3. Long cephalomedullary fixation and radiotherapy
4. Plate and screw hip fixation
5. Plate and screw hip fixation and radiotherapy
Question #127 (Oncology)

Figure 127a Figure 127b Figure 127c


Question #127 (Oncology)

Figures 127a through 127c are the femur radiographs of a 76-year-old man
who is a communityambulator. This man, who has biopsy-proven
metastatic prostate cancer, has been experiencing left hip and thigh pain
while walking for 3 weeks. What is the best next treatment step?

1. Total hip arthroplasty


2. Long cephalomedullary fixation

3. Long cephalomedullary fixation and


radiotherapy
4. Plate and screw hip fixation
5. Plate and screw hip fixation and radiotherapy
Question #127 (Oncology)

• Long Bone Impending or Pathologic Fractures Best Treated with IM


Fixation to provide support to entire bone, and to allow immediate
weight-bearing and function. At approximately 2 weeks or when
wounds are healed, start local Radiotherapy.

• Bickels J, Dadia S, Lidar Z. Surgical management of metastatic bone disease. J Bone Joint Surg Am. 2009
Jun;91(6):1503-16. Review. PubMed PMID:19487532
– IM fixation devices are preferable for pathological fractures because of their superior ability to withstand mechanical loads, because they
support the entire length of the affected bone, and because normal bone-healing cannot be expected
– Cases of extreme cortical destruction in which the remaining cortices cannot support a fixation device, and particularly in which the adjacent
joint is destroyed, should be treated with resection and reconstruction with a cemented tumor prosthesis
– Following wound-healing, patients are referred for adjuvant radiation therapy to decrease the likelihood of local tumor recurrence. This
usually consists of 3000 to 3500 Gy of external beam radiation given in several fractions, usually beginning No earlier than two weeks after
the operation
– After the lungs and liver, the skeleton is the most common site of metastatic disease1,2. Prostate, breast, lung, kidney, and thyroid cancers
account for 80% of all skeletal metastases1-3. The femur, spine, humerus, pelvis, ribs, and skull are reported to be the most commonly
affected sites, in that order

• Damron TA. Treatment principles and prediction of the impending pathologic fracture. In: Schwartz HS,ed. Orthopaedic
Knowledge Update: Musculoskeletal Tumors 2. Rosemont, IL: American Academy of Orthopaedic Surgeons; 2007:369-
374.
Basic
Science Question 147
Q147: Answer
Q147: Explanation

• Bisphosphonates have been shown to be


linked to osteonecrosis of the jaw
• Most commonly affected site is the mandible
followed by the maxilla
• Risk factors
– Age >60
– Female
– Previous invasive dental treatment
Peds/Onc
Question 155

• Figures 155a and 155b are the plain radiographs of a 17-


year-old boy who recently noted painless swelling in his
distal thigh. Examination reveals a firm, fixed, deep distal
thigh mass. There is no associated tenderness. What is the
best next treatment step?
– 1. Biopsy
– 2. Resection
– 3. A CT scan
– 4. An MRI scan
– 5. Observation
Question 155
Question 155

• Figures 155a and 155b are the plain radiographs of a 17-


year-old boy who recently noted painless swelling in his
distal thigh. Examination reveals a firm, fixed, deep distal
thigh mass. There is no associated tenderness. What is the
best next treatment step?
– 1. Biopsy
– 2. Resection
– 3. A CT scan
– 4. An MRI scan
– 5. Observation
Question 155

• Osteochondroma
– Most common bone tumor
– Hamartomatous proliferation of bone & cartilage
– Occur at surface of bone & often at tendon insertion
sites
– Knee, proximal femur, proximal humerus
– Cartilage cap that covers lesion usually 2-3 mm in
thickness
– Need to worry about lesion if growing in adults or
cartilage cap > 2cm
– Possible transformation to chondrosarcoma
– Treatment of painless masses not actively growing
outside of adolescence is observation
Orthopaedic Oncology

Question 170
Figures 170a through 170c are the plain radiographs and coronal short inversion time inversion recovery
(STIR) MRI scan of a 44-year-old woman with metastatic thyroid carcinoma and right shoulder pain. She
reports no history of trauma. Examination reveals no masses and considerable tenderness of the proximal
humerus. What is the best next treatment step?

1. Observation
2. Physical therapy
3. Radiation therapy
4. Prophylactic internal fixation
5. Proximal humeral replacement
Orthopaedic Oncology

Question 170
Figures 170a through 170c are the plain radiographs and coronal short inversion time inversion recovery
(STIR) MRI scan of a 44-year-old woman with metastatic thyroid carcinoma and right shoulder pain. She
reports no history of trauma. Examination reveals no masses and considerable tenderness of the proximal
humerus. What is the best next treatment step?
1. Observation
2. Physical therapy
3. Radiation therapy
4. Prophylactic internal fixation
5. Proximal humeral replacement
Orthopaedic Oncology

Question 170
Figures 170a through 170c are the plain radiographs and coronal short inversion time inversion recovery
(STIR) MRI scan of a 44-year-old woman with metastatic thyroid carcinoma and right shoulder pain. She
reports no history of trauma. Examination reveals no masses and considerable tenderness of the proximal
humerus. What is the best next treatment step?
1. Observation
2. Physical therapy
3. Radiation therapy
4. Prophylactic internal fixation
5. Proximal humeral replacement
Orthopaedic Oncology

Question 170
Figures 170a through 170c are the plain radiographs and coronal short inversion time inversion recovery
(STIR) MRI scan of a 44-year-old woman with metastatic thyroid carcinoma and right shoulder pain. She
reports no history of trauma. Examination reveals no masses and considerable tenderness of the proximal
humerus. What is the best next treatment step?

1. Observation
2. Physical therapy
3. Radiation therapy
4. Prophylactic internal fixation
5. Proximal humeral replacement

Explanation: This question is testing knowledge of Mirels criteria which are based on several factors from the physical examination and
radiographs, MRI was not part of the original Mirels’ score. Here the patient has moderate upper extremity pain with what appears to be a
mixed lesion on X-ray and involvement of <1/3 of the cortex giving an overall Mirels score of 6 and indicating radiotherapy. It is notable
that it has been recommended by Evans et al that the original scoring system be modified so that a score of 7 is use as a threshold for
fixation in the humerus, regardless, in this case the score falls below the threshold and thus radiation therapy is indicated with observation.
Mirels criteria
score > 8 suggests prophylactic fixation
Score 1 2 3
Site   upper limb lower limb peritrochanteric
Pain  mild moderate functional PREFERRED RESPONSE: 3
Lesion  blastic mixed lytic RECOMMENDED READINGS
Evans AR, Bottros J, Grant W, Chen BY, Damron TA. Mirels' rating for humerus
Size  < 1/3 1/3 to 2/3 > 2/3
lesions is both
Clinical reproducible and valid. Clin Orthop Relat Res. 2008 Jun;466(6):1279-84. Epub 2008
Mirels’ score
recommendation Mar 21. PubMed
PMID: 18357496.
Radiotherapy and Mirels H. Metastatic disease in long bones. A proposed scoring system for diagnosing
≤7
observation impending
pathologic fractures. Clin Orthop Relat Res. 1989 Dec;(249):256-64. PubMed PMID:
8 Use clinical judgment 2684463
Jawad et al. Clin Orthop Relat Res. 2010; 468(10): 2825-2827. PubMed PMID:
≥9 Prophylactic fixation 3049613
Question 210

Figures 210a and 210b are the axial T1 and postgadolinium MRI
scans of a 67-year-old woman with a right thigh mass that has
grown in size and has become increasingly symptomatic for 6
months. What is the best next treatment step?

1. Observation
2. Core biopsy
3. Wide resection
4. Marginal resection
5. Radiation therapy
Question 210 Images
Question 210

Figures 210a and 210b are the axial T1 and postgadolinium MRI
scans of a 67-year-old woman with a right thigh mass that has
grown in size and has become increasingly symptomatic for 6
months. What is the best next treatment step?

1. Observation
2. Core biopsy
3. Wide resection
4. Marginal resection
5. Radiation therapy
Question 210

• PREFERRED RESPONSE: 4

• MRI characteristics that support benign lipoma over liposarcoma


include: smaller tumor size (9.4 cm greatest dimension for benign
vs 13.4 cm for malignant), a mass with a uniformly homogeneous
signal (p = 0.0003), a mass with homogeneous high T1 and T2
signals and a low short-time-inversion-recovery (STIR) signal
comparable to normal fat. Needle biopsies are inaccurate and
marginal resection is required when a liposarcoma is present.

• Damron TA. What to do with deep lipomatous tumors. Instr Course


Lect. 2004;53:651-5. Review. PubMed PMID: 15116655.
• Rougraff BT, Durbin M, Lawerence J, Buckwalter K. Histologic
correlation with magnetic resonance imaging for benign and
malignant lipomatous masses. Sarcoma. 1997;1(3-4):175-9.
PubMed PMID: 18521221.
Oncology
Question 220

• Figures 220a through 220c are the radiograph,


MRI scan, and biopsy specimen of a 5-year-old
boy with a 4-week history of right hip pain, limp,
and a low-grade fever. He also has diabetes
insipidus, exopthalmus, and multiple lesions in
the skull. What is the most likely diagnosis?

• 1. Ollier disease
• 2. Gorham disease
• 3. Hand-Schuller-Christian disease
• 4. Mazabraud syndrome
• 5. McCune-Albright syndrome
Question 220
Question 220

• Figures 220a through 220c are the radiograph,


MRI scan, and biopsy specimen of a 5-year-old
boy with a 4-week history of right hip pain, limp,
and a low-grade fever. He also has diabetes
insipidus, exopthalmus, and multiple lesions in
the skull. What is the most likely diagnosis?

• 1. Ollier disease
• 2. Gorham disease
• 3. Hand-Schuller-Christian disease
• 4. Mazabraud syndrome
• 5. McCune-Albright syndrome
Question 220
• Hand-Schuller-Christian (HSC) disease is a chronic,
disseminated form of Langerhans’ cell histiocytosis with
bone and visceral involvement. The classic triad of HSC
includes multiple lytic skull lesions, diabetes insipidus, and
exopthalmos. MRI appearance: soft tissue mass adjacent to
boney lesions. Histologic appearance: mononuclear
histiocyte-like cells with oval nuclei and well-defined round
or oval eosinophilic cytoplasm.

• RECOMMENDED READINGS
• Pitcher JD Jr, Weber KL. Benign fibrous and histiocytic lesions. In: Schwartz HS, ed.
Orthopaedic Knowledge Update: Musculoskeletal Tumors 2. Rosemont, IL: American
Academy of Orthopaedic Surgeons; 2007:121-132.
• Velez-Yanguas MC, Warrier RP. Langerhans’ cell histiocytosis. Orthop Clin North Am.
1996 Jul;27(3):615-23. Review. PubMed PMID: 8649742.
• Islinger RB, Kuklo TR, Owens BD, Horan PJ, Choma TJ, Murphey MD, Temple HT.
Langerhans’ cell histiocytosis in patients older than 21 years. Clin Orthop Relat Res.
2000 Oct;(379):231-5. PubMed PMID: 11039811.
233

• Figures 233a through 233c are the radiographs


and biopsy specimen of a 32-year-old woman
who has had
• progressive ankle pain for 6 months. What is the
most appropriate treatment option?
• 1. Radiotherapy alone
• 2. Intralesional curettage
• 3. Intralesional curettage with adjuvants
• 4. Wide surgical resection and radiotherapy
• 5. Neoadjuvant chemotherapy and wide
resection
233
233

• Figures 233a through 233c are the radiographs


and biopsy specimen of a 32-year-old woman
who has had
• progressive ankle pain for 6 months. What is the
most appropriate treatment option?
• 1. Radiotherapy alone
• 2. Intralesional curettage
• 3. Intralesional curettage with adjuvants
• 4. Wide surgical resection and radiotherapy
• 5. Neoadjuvant chemotherapy and wide
resection
233
• McDonald DJ, Weber KL. Giant cell tumor of bone. In: Schwartz HS, ed. Orthopaedic Knowledge
Update: Musculoskeletal Tumors 2. Rosemont, IL: American Academy of Orthopaedic Surgeons;
2007:133-140.
• Arbeitsgemeinschaft Knochentumoren. Local recurrence of giant cell tumor of bone after
intralesional treatment with and without adjuvant therapy. J Bone Joint Surg Am. 2008
May;90(5):1060-7. PubMed PMID:

• Giant Cell Tumor; benign aggressive tumor typically found in the epiphysis


of long bones
• F>M, 30-50 years old
• Eccentric lytic epiphyseal/metaphyseal lesion that often extends into the
distal epiphysis and borders subchondral bone 
• Neoplastic cell is the mononucleur stromal cell, hallmark giant cells are
numerous 
• Treatment is curettage with cementing
• Hand lesions controversial: may require amputation
242
242
242
242
• All patients who have a proximal femoral replacement have
abductor weakness, which is the reason the answer 2. This is an
expected outcome, not a complication.
• The referenced paper by Dr. Parvizi (which is a review of the
literature) states:

• This suggests that the most common complication is instability.


Tumor

267

• Figures 267a through 267c are the radiographs and biopsy


specimen of a 10-year-old boy who is experiencing lateral ankle
pain during sports and recreational activities. What is the best
next treatment step?
• 1. Wide resection
• 2. Curettage and bone graft
• 3. Aspiration and steroid injection
• 4. Chemotherapy and wide resection
• 5. Observation and activity restriction
267
267

• Figures 267a through 267c are the radiographs and biopsy


specimen of a 10-year-old boy who is experiencing lateral ankle
pain during sports and recreational activities. What is the best
next treatment step?
• 1. Wide resection
• 2. Curettage and bone graft
• 3. Aspiration and steroid injection
• 4. Chemotherapy and wide resection
• 5. Observation and activity restriction
267
The lesion is an Aneurysmal bone cyst (ABC), which is a benign and non-neoplastic reactive bone
lesion filled with multiple blood-filled cavities. It can be locally destructive to normal bone and
may extend to soft tissue. Most commonly found in pediatric patients. Radiographically
appears as expansile, eccentric and lytic lesion with bony septae ("bubbly appearance").
Usually in the metaphysis. On CT or MRI, multiple fluid levels can be observed. Caracteristic
histological appearance is a blood filled space with an endothelial lining, giant cells can be
observed. If the lesion is discovered without acute fracture, the treatment is aggressive
curretage with bone grafting. Recurrence rate is up to 25%. In the setting of acute fracture,
manage conservatively as the fracture may result in spontaneous healing of the ABC.

The treatment for an ABC is curretage and grafting.

• RECOMMENDED READINGS
• Weber KL, Heck RK Jr. Cystic and benign bone lesions. In: Schwartz HS, ed. Orthopaedic Knowledge Update:
Musculoskeletal Tumors 2. Rosemont, IL: American Academy of Orthopaedic Surgeons; 2007:87-102.
• Greenspan A, Jundt G, Remagen W, eds. Differential Diagnosis In Orthopaedic Oncology. 2nd ed. Philadelphia, PA:
Lippincott Williams & Wilkins; 2007:387-457.
• Steffner RJ, Liao C, Stacy G, Atanda A, Attar S, Avedian R, Peabody TD. Factors associated with recurrence of primary
aneurysmal bone cysts: is argon beam coagulation an effective adjuvant treatment? J Bone Joint Surg Am. 2011 Nov
2;93(21):e1221-9. PubMed PMID: 22048101.
Question 4
A 62-year-old otherwise healthy man underwent resection of a mass in his
anterior thigh. The 8-cm superficial mass was a high-grade pleomorphic
sarcoma, but no margins could be determined. The surgeon thought the mass
was a lipoma, but stated that adequate margins were identified intrasurgically.
He did not send frozen section margins. Which treatment would best improve
local control?
1. Observation
2. Radiation to the surgical bed
3. Wide re-resection of the surgical bed and radiation
4. Chemotherapy and re-resection of the surgical bed
5. Wide re-resection of the surgical bed
Question 4
A 62-year-old otherwise healthy man underwent resection of a mass in his
anterior thigh. The 8-cm superficial mass was a high-grade pleomorphic
sarcoma, but no margins could be determined. The surgeon thought the mass
was a lipoma, but stated that adequate margins were identified intrasurgically.
He did not send frozen section margins. Which treatment would best improve
local control?
1. Observation
2. Radiation to the surgical bed
3. Wide re-resection of the surgical bed and radiation
4. Chemotherapy and re-resection of the surgical bed
5. Wide re-resection of the surgical bed
• Advise repeat excision for all patients who have had an unplanned excision of
soft tissue sarcoma of an extremity
• 1/3 patients had residual tumor at site, but cannot predict who
• Allows surgeon and oncologist to then characterize the residual tumor burden
• Then, in consultation with radiation oncologist, may decide for radiation as well
depending on post operative margins
• May also consider preoperative radiation in some cases
• Attempt to review histology to look for margins from initial surgery if possible
• Observation if can clearly document that initial resection was radical
(histologically)
• If initial resection margins clear of disease, but not radical excision, can do
radiation alone
• After re-resection, if there is residual disease within 2cm of margins, post op
radiation generally recommended
• TAKE HOME: if do not have clear negative margins from initial unplanned soft
tissue sarcoma resection, need wide re-resection of surgical bed +/- radiation

Venkatesan M, Richards CJ, McCulloch TA, Perks AG, Raurell A, Ashford RU; East Midlands Sarcoma Service. Inadvertent
surgical resection of soft tissue sarcomas. Eur J Surg Oncol. 2012 Apr;38(4):346-51. doi: 10.1016/j.ejso.2011.12.011. Epub
2012 Jan 20. PubMed PMID: 22264775.
Noria S, Davis A, Kandel R, Levesque J, O’Sullivan B, Wunder J, Bell R. Residual disease following unplanned excision of soft-
tissue sarcoma of an extremity. J Bone Joint Surg Am. 1996 May;78(5):650-5. PubMed PMID: 8642020.
Question 17
•Figures 17a through 17d are anteroposterior and
lateral radiographs of and low- and high-power
biopsy specimens of a 67 year old woman who has
progressive left hip pain. She describes pain at rest
that is exacerbated with weight bearing. What is
the next best step?
1. Observation
2. Wide resection
3. Internal fixation
4. Chemotherapy
5. Radiation therapy
Question 17
•Figures 17a through 17d are anteroposterior and
lateral radiographs of and low- and high-power
biopsy specimens of a 67 year old woman who has
progressive left hip pain. She describes pain at rest
that is exacerbated with weight bearing. What is
the next best step?
1. Observation
2. Wide resection
3. Internal fixation
4. Chemotherapy
5. Radiation therapy
Question 17
•Treatment of metastatic bone disease is guided by the nature of the
skeletal related event, the responsiveness of the lesion to adjuvant
care, and the overall condition and survival expectation of the
patient. Pathologic fractures are an important cause of morbidity
and mortality in patients with metastatic bone disease. Pathologic
fractures have a diminished ability to heal spontaneously. Fracture
stabilization with internal fixation or arthroplasty may substantially
improve patient mobility and quality of life. In the lower extremity
and spine, internal fixation should be performed in most patients
expected to survive another six to twelve weeks. Although
morbidity and even mortality (8% for total hip replacement) can be
high, intervention substantially improves the remaining quality of
life.

Robert H. Quinn, MD; R. Lor Randall, MD; Joseph Benevenia, MD; Sigurd H. Berven, MD; Kevin A.
Raskin, MD. J Bone Joint Surg Am, 2013 Oct 16; 95 (20): 1887 -1895 .
25
• Figures 25a through 25d are the radiographs and
biopsy specimens of a 20-year-old man who has
ambulatory pain in his right heel. What is the most
likely diagnosis?

• 1. Chondroblastoma
• 2. Intraosseous lipoma
• 3. Unicameral bone cyst
• 4. Aneurysmal bone cyst
• 5. Giant-cell tumor of bone
Question 25
• Figures 25a through 25d are the radiographs and
biopsy specimens of a 20-year-old man who has
ambulatory pain in his right heel. What is the most
likely diagnosis?

• 1. Chondroblastoma
• 2. Intraosseous lipoma
• 3. Unicameral bone cyst
• 4. Aneurysmal bone cyst
• 5. Giant-cell tumor of bone
Question 25
• UBC = Cyst with thin fibrous lining, no osseous reaction
• Uniform population of spindle cells without nuclear atypia
• No histological findings of cartilaginous cells, fat cells, blood
cysts, or Giant cells

• Kilgore WB, Parrish WM. Calcaneal tumors and tumor-like


conditions. Foot Ankle Clin. 2005 Sep;10(3):541-65, vii.
Review. PubMed PMID: 16081020.
• Weber KL, Heck RK: Cystic and benign bone lesions. In:
Schwartz HS, ed. Orthopaedic Knowledge Update:
Musculoskeletal Tumors 2. Rosemont, IL: American
Academy of Orthopaedic Surgeons;2007:87-102.
Question #37
Answer #37

Classic presentation for Osteosarcoma. Normally you don’t want to


fall into the trap of jumping to a treatment without a definitive diagnosis
but in this case the “best next step” is the chemo-surgery-chemo
sandwich. Answer is 2-3 months or preoperative chemo (as in this
case), followed by surgery, then a more protracted course of chemo.
Question 47
•Figures 47a through 47c are the radiographs of a 65-year-
old man with a history of lung adenocarcinoma. He has thigh
pain with weight bearing. His bone scan findings reveal no
other osseous lesions. What is the most appropriate next
step?
1. Biopsy
2. Radiotherapy
3. Total femur replacement
4. Intramedullary nail and radiotherapy
5. Wide resection and intercalary reconstruction
Question 47
Question 47
•Figures 47a through 47c are the radiographs of a 65-
year-old man with a history of lung adenocarcinoma. He
has thigh pain with weight bearing. His bone scan findings
reveal no other osseous lesions. What is the most
appropriate next step?
1. Biopsy
2. Radiotherapy
3. Total femur replacement
4. Intramedullary nail and radiotherapy
5. Wide resection and intercalary reconstruction
Bone metastases
•Operative treatment of metastatic bone disease cannot
be carried out without an established histological
diagnosis
•When a patient has no previous histological diagnosis of
metastatic bone disease, a biopsy is required to establish
the diagnosis and exclude tumors that predictably respond
to nonoperative treatment (i.e. lymphoma) or that require
a different treatment strategy (i.e.sarcoma)
•Local tumor control is usually accomplished by post-
operative radiation therapy
Question 60
Question 60
Explanation

Damron et al:
Biopsy: for soft tissue mass when clinical/radiopgrahic eval
does not yield conclusive dx or when when mass must be removed,
do NOT use transverse incision, no other procedures performed at
same time, can send fluid from any collection for analysis
Before proceeding: Need MSK pathologist, and surgeon with
experience to deal with all of possible dx of differential; imaging
should be done before bx

*Negative FNA/core needle: get more tissue


*Open bx: OK for tourniquet, Gravity exsanguination only,
collect sample without crushing/cautery and send for frozen,
hemostasis after tourniquet release to avoid contamination by
dissection of bleeding elsewhere in extremity. Drain to be placed
with exit in line/close to end of incision
Question #79
Answer #79

Two clinical entities (osteofibrous dysplasia and adamantinoma) have


almost identical presentations from age (<20), anatomic location
(tibia), radiographic appearance, to sudden onset of pain/swelling in
the limb with up to 30% presenting with a pathologic fracture. Notably
these are both exceedingly rare clinical entities. What allows you to
differentiate the two is the histology. Adamantinomas are epithelial
derived, thus do not “usually” have spicules of bone (and most likely
won’t on the OITE). Thus if you see islands of bone and specifically if
that bone is lined with the characteristically present osteoblasts then
you’ve got an Osteofibrous Dysplasia (aka OFD). See next slide for
representative histo slides.
Question #79
Answer #79

Two clinical entities (osteofibrous dysplasia and adamantinoma) have


almost identical presentations from age (<20), anatomic location
(tibia), radiographic appearance, to sudden onset of pain/swelling in
the limb with up to 30% presenting with a pathologic fracture. Notably
these are both exceedingly rare clinical entities. What allows you to
differentiate the two is the histology. Adamantinomas are epithelial
derived, thus do not “usually” have spicules of bone (and most likely
won’t on the OITE). Thus if you see islands of bone and specifically if
that bone is lined with the characteristically present osteoblasts then
you’ve got an Osteofibrous Dysplasia (aka OFD). See next slide for
representative histo slides.
Question 90
Figures 90a through 90c are the radiographs and pathology of a 20-year-old
man who has knee pain and swelling 1 year after undergoing curetting of a
benign bone tumor of the proximal tibia. What is the most likely diagnosis?

1. Osteomyelitis

2. Chondroblastoma

3. Aneurysmal bone cyst

4. Giant-cell tumor of bone

5. Malignant giant-cell tumor


Question 90
Explanation
4. Giant-cell tumor of bone
–Benign aggressive tumor found mostly on distal
femur > prox tibia > distal radius > sacral ala. 50%
of them occur around knee. Pain referred to
involved joint.
–If treated only with curettage, it has a 10-30%
chance of recurring. Only 3% chance if combined
with adjuvant chemo.
–Path shows giant cells, in the right location.
Suggested Reading

•Raskin KA, Schwab JH, Mankin HJ, Springfield DS, Hornicek FJ. Giant cell
tumor of bone. J Am Acad Orthop Surg. 2013 Feb;21(2):118-26. doi:
10.5435/JAAOS-21-02-118. Review. PubMed PMID: 23378375.

•McDonald DJ, Weber KL: Giant cell tumor of bone. In: Schwartz HS, ed.
Orthopaedic Knowledge Update: Musculoskeletal Tumors 2. Rosemont, IL:
American Academy of Orthopaedic Surgeons;2007:133-140.

•Becker WT, Dohle J, Bernd L, Braun A, Cserhati M, Enderle A, Hovy L,


Matejovsky Z, Szendroi M, Trieb K, Tunn PU. Local recurrence of giant cell
tumor of bone after intralesional treatment with and without adjuvant
hterapy. Lancet Oncol. 2010; 11(3): 275-280.
Question 109
• Figures 109a and 109b are the radiographs of a 66-
year-old woman with metastatic breast cancer who has
a 3- to 4-week history of progressively worsening pain
in her left leg and an inability to bear weight on her left
lower extremity. Her estimated survival time is longer
than 6 months. What is the appropriate next step?
• 1. Biopsy of the area
• 2. Intramedullary nail fixation
• 3. Consultation with hospice care
• 4. Consultation with a radiation oncologist
• 5. Consultation with a chemotherapy oncologist
109
• Figures 109a and 109b are the radiographs of a 66-
year-old woman with metastatic breast cancer who has
a 3- to 4-week history of progressively worsening pain
in her left leg and an inability to bear weight on her left
lower extremity. Her estimated survival time is longer
than 6 months. What is the appropriate next step?
• 1. Biopsy of the area
• 2. Intramedullary nail fixation
• 3. Consultation with hospice care
• 4. Consultation with a radiation oncologist
• 5. Consultation with a chemotherapy oncologist
Question 109
• Prophylactic fixation of impending WB fracture gold
standard
• Proven primary cancer
• Life expectancy > 6 months, so hospice not viable option

• Biermann JS, Holt GE, Lewis VO, Schwartz HS, Yaszemski


MJ. Metastatic bone disease: diagnosis, evaluation, and
treatment. J Bone Joint Surg Am. 2009 Jun;91(6):1518-
30. Review. PubMed PMID: 19487533.
• Coleman RE. Skeletal complications of malignancy.
Cancer. 1997 Oct 15;80(8 Suppl):1588-94. Review.
PubMed PMID: 9362426.
Question 127

Question 127: Figures 127a through 127c are the axial T1, coronal short tau
inversion recovery, and axial postcontrast MR images of a 49-year-old woman who
has an enlarging, increasingly symptomatic posterior arm mass. She is not sure
how long the mass has been present. What is the most likely diagnosis?

•1. Lipoma

•2. Myxoma

•3. Hemangioma

•4. Atypical lipoma

•5. Soft-tissue sarcoma


Question 127
Question 127: Figures 127a through
127c are the axial T1, coronal short
tau inversion recovery, and axial
postcontrast MR images of a 49-
year-old woman who has an
enlarging, increasingly symptomatic
posterior arm mass. She is not sure
how long the mass has been present.
What is the most likely diagnosis?

•1. Lipoma

•2. Myxoma

•3. Hemangioma

•4. Atypical lipoma

•5. Soft-tissue sarcoma


Question 127

Simple lipomas and well-differentiated liposarcomas are both grossly fatty masses.
MRI has been described as useful in attempting to distinguish these two lesions.
Features more closely associated with well-differentiated liposarcoma: thickened or
nodular septa (>2mm), associated nonadipose masses, prominent foci of high T2
signal, and prominent areas of enhancement. Simple lipomas may contain a few thin,
discrete septa but are otherwise homogeneously fatty masses. Higher grade
liposarcomas typically contain little or no macroscopic fat.

Gaskin CM, Helms CA. Lipomas, lipoma variants, and well-differentiated liposarcomas (atypical lipomas): results
of MRI evaluations of 126 consecutive fatty masses. AJR Am J Roentgenol. 2004 Mar;182(3):733-9.
PMID: 14975977
Question 156
Figures 156a and 156b are the plain radiograph and biopsy specimen of a 14-year-old
boy who has a 6-week history of right leg pain. The most appropriate next diagnostic
steps should include
1. a bone scan, MR of the femur, and CT of the chest.
2. a bone scan; MR of the femur; and CT of the chest, abdomen, and pelvis.
3. a bone marrow aspirate; CT of the chest, abdomen, and pelvis; and a bone scan.
4. a bone marrow biopsy; CT of the chest, abdomen, and pelvis; MR of the femur; and
a bone scan.
5. nothing further; a biopsy should be adequate to stage this disease.
Question 156
Question 156
Figures 156a and 156b are the plain radiograph and biopsy specimen of a 14-year-old
boy who has a 6-week history of right leg pain. The most appropriate next diagnostic
steps should include
1. a bone scan, MR of the femur, and CT of the chest.
2. a bone scan; MR of the femur; and CT of the chest, abdomen, and pelvis.
3. a bone marrow aspirate; CT of the chest, abdomen, and pelvis; and a bone scan.
4. a bone marrow biopsy; CT of the chest, abdomen, and pelvis; MR of the femur; and
a bone scan.
5. nothing further; a biopsy should be adequate to stage this disease.
Osteosarcoma
• Most common primary bone sarcoma
• Bimodal age distribution
– 2nd decade of life
– Elderly patients with Paget’s Disease
• Most commonly presents in distal femur and proximal tibia
• Can also present in proximal humerus, proximal femur, and pelvis
• 10-20% patients present with pulmonary metastasis (l ung is most common site of
metastasis)
• 76% long term survival
• Imaging:
– Radiographs may show blastic and destructive lesion (sunburst or hair on end appearance)
– MRI of whole bone to assess soft tissue involvement, neurovascular involvement, and skip
metastases
– Bone scan hot in osteosarcoma, assess bone metastasis
– CT chest to evaluate for pulmonary metastasis

Messerschmitt PJ et al. Osteosarcoma. JAAOS, 2009 Aug;17(8):515-27


Question 172
Figures 172a and 172b are the left humerus radiographs of a 78-year-old woman with
multiple myeloma and multifocal osseous disease who has progressive left arm pain and rest
and with activity. Comprehensive care of this patient’s osseous disease should include
1.Radiation to the left humerus and bisphosphonates
2.Surgical stabilization and radiation to the left humerus
3.Surgical stabilization and radiation to the left humerus and bisphosphonates
4.Surgical stabilization and radiation to the left humerus, bisphosphonates, and hormone
therapy
5.Surgical stabilization and bisphosphonates
Question 172
Figures 172a and 172b are the left humerus radiographs of a 78-year-old woman with
multiple myeloma and multifocal osseous disease who has progressive left arm pain and rest
and with activity. Comprehensive care of this patient’s osseous disease should include
1.Radiation to the left humerus and bisphosphonates
2.Surgical stabilization and radiation to the left humerus
3.Surgical stabilization and radiation to the left humerus and bisphosphonates
4.Surgical stabilization and radiation to the left humerus, bisphosphonates, and hormone
therapy
5.Surgical stabilization and bisphosphonates
• If already have a diagnosis of MM, no further biopsy of the lesion is required for
diagnosis
• When the tumor is the cause of pain, the lesion frequently responds to radiation
therapy
• Nonsurgical management should be considered for lesions in non-weightbearing
areas when surgery could delay systemic care
• In patients with multiple myeloma, surgical indications are typically limited to
management of impending or pathologic fractures
• Basic principles of surgery for patients with metastatic disease: preoperative
imaging of entire involved bone, achieving immediately stable and durable
construct, postoperative radiation
• Monthly IV infusions of either pamidronate or zoledronic acid (bisphosphonates)
have reduced skeletal complications among patients with MM and are now a
mainstay of myeloma therapy
• Orally administered bisphosphonates have shown little benefit
• TAKE HOME: Comprehensive osseous care in a patient with MM and
impending pathologic fx includes surgical stabilization, radiation, and IV
bisphosphonates
Scharschmidt TJ, Lindsey JD, Becker PS, Conrad EU. Multiple myeloma: diagnosis and orthopaedic implications. J Am Acad Orthop Surg. 2011 Jul;19(7):410-9.

Review. PubMed PMID: 21724920.

Yeh HS, Berenson JR. Treatment for myeloma bone disease. Clin Cancer Res. 2006 Oct 15;12(20 Pt 2):6279s-6284s. Review. PubMed PMID: 17062714.
Question 187
Figures 187a through 187f are the plain radiographs, coronal short
tau inversion recovery and axial T1 MR images, and biopsy
specimens of a 9-year-old boy who has had a limp for 4 months.
He has intermittent right hip pain and mild irritability around the
hip. What is the most likely diagnosis?

1.Fibrous dysplasia
2.Osteofibrous dysplasia
3.Unicameral bone cyst
4.Aneurysmal bone cyst
5.Eosinophilic granuloma
Question 187
Question 187
Figures 187a through 187f are the plain radiographs, coronal
short tau inversion recovery and axial T1 MR images, and biopsy
specimens of a 9-year-old boy who has had a limp for 4 months.
He has intermittent right hip pain and mild irritability around the
hip. What is the most likely diagnosis?
1.Fibrous dysplasia
2.Osteofibrous dysplasia
3.Unicameral bone cyst
4.Aneurysmal bone cyst
5.Eosinophilic granuloma
Unicameral bone cysts (UBC)
•Unicameral bone cysts are benign bone tumors that occur in children
and adolescents. They commonly affect long bones (radius, proximal
humerus and femur) near the physis.. Radiographic findings include
symmetric cystic expansion with thinning corticies. Pain results from
pathologic fracture.
•Pathology findings: thin, fibrous lining contains fibrous tissue, giant cells,
hemosiderin pigment, and a few chronic inflammatory cells.
•Treatment: aspiration to confirm the diagnosis, followed by
methylprednisolone acetate injection.
•Unicameral bone cysts of the proximal femur are often treated with
curettage, grafting, and internal fixation to avoid fracture and
osteonecrosis.

Weber KL, Heck RK Jr. Cystic and benign bone lesions. In: Schwartz HS, ed. Orthopaedic Knowledge Update: Musculoskeletal Tumors 2.
Rosemont, IL: American Academy of Orthopaedic Surgeons;2007:89-92.
Question 217
Figures 217a through 217e are the plain radiographs, coronal and axial T2-weighted
MR images, and
biopsy specimen of a 16-year-old boy who has severe activity-related knee pain. He
has a firm, fixed, deep
distal thigh mass. What is the most likely diagnosis?

1. Osteosarcoma
2. Osteoblastoma
3. Chondrosarcoma
4. Osteochondroma
5. Nonossifying fibroma
Question 217
Figures 217a through 217e are the plain radiographs, coronal and axial T2-weighted
MR images, and
biopsy specimen of a 16-year-old boy who has severe activity-related knee pain. He
has a firm, fixed, deep
distal thigh mass. What is the most likely diagnosis?

1. Osteosarcoma
2. Osteoblastoma
3. Chondrosarcoma
4. Osteochondroma
5. Nonossifying fibroma
Osteochondroma/MHE
A benign chondrogenic lesion derived from aberrant cartilage from the perichondral
ring. May be solitary or multiple (MHE). The most common benign bone tumor.
Common in adolescents and young adults. Occur on the surface of the bone often at
sites of tendon insertion. Common locations are prox tibia, distal femur, prox femur,
subungual (most common at the hallux). MHE is inherited in an autosomal dominant
fashion from a mutation in the EXT gene. Osteochondroma has the potential to
become chondrosarcoma (<1% with solitary, 5-10% with MHE), most commonly in the
pelvis. Present as a painless mass. Histology is similar to normal physis. Surgical
resection decision is made based on symptoms.

Temple HT, Scully SP, Aboulafia AJ. Benign bone tumors. Instr Course Lect. 2002;51:429-39.
Review. PubMed PMID: 12064131.
Stieber JR, Dormans JP. Manifestations of hereditary multiple exostoses. J Am Acad Orthop Surg.
2005 Mar-Apr;13(2):110-20. Review. PubMed PMID: 15850368.
Question 241
Figures 241a and 241b are the MR images of the right hip and the bone scan
of a 50-year-old man who has progressively worsening right hip pain and
difficulty ambulating. What is the best next step?

1. Consultation with a medical oncologist


2. Consultation with a radiation oncologist
3. Surgical stabilization or hemiarthroplasty of the area
4. Biopsy and surgical stabilization of the area while in the operating room
5. CT scans of the chest, abdomen, and pelvis followed by biopsy
Figures 241A and 241B
Question 241
Figures 241a and 241b are the MR images of the right hip and the bone scan
of a 50-year-old man who has progressively worsening right hip pain and
difficulty ambulating. What is the best next step?

1. Consultation with a medical oncologist


2. Consultation with a radiation oncologist
3. Surgical stabilization or hemiarthroplasty of the area
4. Biopsy and surgical stabilization of the area while in the operating room
5. CT scans of the chest, abdomen, and pelvis followed by biopsy
Question 241
The concern, given patients age and location, is for a metastatic lesion to the proximal femur. You would
first perform a work up consisting of CT C/A/P looking for primary source, or other metastatic lesions.
You would then perform a biopsy to confirm carcinoma vs primary sarcoma.

Rougraff et al, performed a similar diagnositic study of 40 consecutive patients. This included recording
of a medical history; physical examination; routine laboratory analysis; plain radiography of the involved
bone and the chest; whole-body technetium-99m-phosphonate bone scintigraphy; and computed
tomography of the chest, abdomen, and pelvis. After this evaluation, a biopsy of the most accessible
osseous lesion was done. Lab values were no specific in all patients. The H&P revealed the occult
primary site of the malignant tumor in three patients (8%). Plain CXRs established the diagnosis of
carcinoma of the lung in seventeen patients (43%). Computed tomography of the chest identified an
additional six primary carcinomas of the lung (15%). Computed tomography of the abdomen and pelvis
established the diagnosis in five patients (13%). Examination of the biopsy tissue established the
diagnosis in only three additional patients (8%) and confirmed it in eleven others. On the basis of the
biopsy alone, we were unable to identify the primary site of the malignant tumor in twenty-six (65%) of
the patients.

•Rougraff BT. Evaluation of the patient with carcinoma of unknown origin metastatic to bone. Clin Orthop Relat Res. 2003 Oct;(415
Suppl):S105-9. Review.PubMed PMID: 14600599.
•Rougraff BT, Kneisl JS, Simon MA. Skeletal metastases of unknown origin. A prospective study of a diagnostic strategy. J Bone Joint Surg Am.
1993 Sep;75(9):1276-81. PubMed PMID: 8408149.
Question 254
• Figures 254a and 254b are the MR image and plain
radiograph of a 32-year-old man with a several-year
history of right knee pain that includes intermittent
swelling and warmth in the knee region. An open
biopsy hematoxylin and eosin stain is shown in Figure
254. What is the best next step?
– 1. Wide resection alone
– 2. Intravenous antibiotic therapy and curretage
– 3. Chemotherapy and wide resection
– 4. Chemotherapy, radiation, and wide resection
– 5. Surgical stabilization with an intramedullary nail
Question 254
Question 254
• Figures 254a and 254b are the MR image and plain
radiograph of a 32-year-old man with a several-year
history of right knee pain that includes intermittent
swelling and warmth in the knee region. An open
biopsy hematoxylin and eosin stain is shown in Figure
254. What is the best next step?
– 1. Wide resection alone
– 2. Intravenous antibiotic therapy and curretage
– 3. Chemotherapy and wide resection
– 4. Chemotherapy, radiation, and wide resection
– 5. Surgical stabilization with an intramedullary nail
Question 254
• Brodie’s abscess
– Subactue pyogenic osteomyelitis
– Often found in the metaphysis of the tibia
– Respond well to irrigation, debridement,
curettage, & IV antibiotics
– H&E stain full of lymphocytes no cells with
abnormal appearing nuclei
– Inflammation round bone on MRI w/ pathology
being bright on T2 like fluid
Question 257
Figures 257a through 257e are the MR images and biopsy
specimens of a 75-year-old woman who has an enlarging
posterior thigh mass. She developed the mass after a fall
several months ago. Examination reveals she has a firm,
fixed, deep mass. What is the most likely diagnosis?
1. Myxoma
2. Hematoma
3. Fibromatosis
4. Neurofibroma
5. Soft-tissue sarcoma
Question 257
Question 257
Figures 257a through 257e are the MR images and biopsy
specimens of a 75-year-old woman who has an enlarging
posterior thigh mass. She developed the mass after a fall
several months ago. Examination reveals she has a firm,
fixed, deep mass. What is the most likely diagnosis?
1. Myxoma
2. Hematoma
3. Fibromatosis
4. Neurofibroma
5. Soft-tissue sarcoma
Question 257
•Soft tissue sarcoma: non-tender, firm, well-circumscribed. Often history of
trauma that is likely incidental. MRI with low signal intensity on T1, high signal
intensity on T2, a heterogeneous appearance, and contrast enhancement are
specific MRI common to soft-tissue sarcomas
•Neurofibroma: superficial, slow-growing and painless. Histology with interlacing
bundles of elongated cells with wavy, dark-staining nuclei. Cells are associated with
wire-like strands of collagen.
•Fibromatosis: extraabdominal desmoid tumor (most locally invasive of all benign
soft tissue tumors. Distinctive “rock hard” character. Histology with well-
differentiated fibroblasts and abundant collagen. Lesion infiltrates adjacent tissues
•Hematoma: may occur after trauma to the extremity. Organizes and resolves
with time.
•Myxoma: poorly circumscribed, merges with surrounding tissue. Histology with
spindle cells. Stroma with mucin and collagen.
Question 262
•Figures 262a through 262e are the plain radiographs, axial
T2-weighted MR image, and low- and high- power biopsy
specimens of a 17-year-old boy who has had progressive
ankle pain following a baseball injury. Examination reveals
diffuse tenderness around the medial hindfoot. What is the
most likely diagnosis?

•Giant-cell tumor
•Fibrous dysplasia
•Chondroblastoma
•Unicameral bone cyst
•Eosinophilic granuloma
Question 262
•Figures 262a through 262e are the plain radiographs, axial
T2-weighted MR image, and low- and high- power biopsy
specimens of a 17-year-old boy who has had progressive
ankle pain following a baseball injury. Examination reveals
diffuse tenderness around the medial hindfoot. What is the
most likely diagnosis?

•Giant-cell tumor
•Fibrous dysplasia
•Chondroblastoma
•Unicameral bone cyst
•Eosinophilic granuloma
Question 262
•The patient has an osseus/cartlilagenous lesion on his medial talus. A
UBC and fibrous dysplasia would be confined within the bone and
occasional cortical destruction would also be seen with a GCT. Pathology
shows chondroblasts in a “chicken-wire” or “cobblestone” pattern
consistent with chondroblastoma. Treatment is intralesional curettage
and bone grafting as this lesion in benign. CT scan of the chest is indicated
to look for pulmonary mets.

•Srinivasan R, Parvin SD, Lambert D. Spontaneously ruptured middle colic


artery aneurysm in a patient with Marfan’s syndrome. Eur J Vasc Surg.
1990 Jun;4(3):317-8. PubMed PMID: 2354728.
Douis H, Saifuddin A. The imaging of cartilaginous bone tumours. I. Benign
lesions. Skeletal Radiol. 2012 Sep;41(10):1195-212. doi:10.1007/s00256-
012-1427-0. Epub 2012 Jun 17. Review. PubMed PMID: 22707094.
Question 266
Figures 266a through 266d are the plain radiographs and biopsy
specimens of a 48-year-old woman who has progressive wrist
sprain and swelling. She has had considerable activity-related
pain, and now has pain that awakens her from sleep.
•What is the most likely diagnosis?
1. Brown tumor
2. Osteosarcoma
3. Giant-cell tumor
4. Chondroblastoma
5. Metastatic carcinoma
266c

266a 266b
266d
Question 266
Figures 266a through 266d are the plain radiographs and biopsy
specimens of a 48-year-old woman who has progressive wrist
sprain and swelling. She has had considerable activity-related
pain, and now has pain that awakens her from sleep.
•What is the most likely diagnosis?
1. Brown tumor
2. Osteosarcoma
3. Giant-cell tumor
4. Chondroblastoma
5. Metastatic carcinoma
Question 266
Giant Cell Tumor
• A benign mesenchymal tumor that has characteristic multinuclear giant
cells. Mononuclear stromal cells are the physiologically active and
diagnostic cell type.
• Most GCTs are located in the epiphyseal regions of long bones.
• Present with pain, swelling, joint effusion, and disability in the third and
fourth decades of life.
• Typically, have well-defined radiographic borders.
• Extended intralesional curettage with or without adjuvant therapy is the
primary treatment choice.

Raskin KA, Schwab JH, Mankin HJ, Springfield DS, Hornicek FJ. Giant cell tumor of bone. J Am Acad
Orthop Surg. 2013 Feb;21(2):118-26. doi: 10.5435/JAAOS-21-02-118. Review. PubMed PMID: 23378375.
McDonald DJ, Weber KL. Giant cell tumor of bone. In: Schwartz HS, ed. Orthopaedic Knowledge Update: Musculoskeletal
Tumors 2. Rosemont, IL: American Academy of Orthopaedic Surgeons; 2007:133-140.
Question 270
Question 270
Explanation
Lewin et al: Review, 2013
Giant cell tumor of bone: osteolytic, usually benign. Infiltration with
osteoclast-like giant cells. RANKL (osteoclast differentiation factor) is heavily
involved in pathogenesis.
Denosumab: inhib RANKL; FDA approved for treatment of giant cell
tumor of bone that is unresectable or when surgery is likely to result in severe
morbidity. Unclear the optimal scheduling, patient selection, adjuvant use,
and long-term toxicity.
.

Xu et al: Review, 2013


Giant cell tumor of bone: no available treatment option definitively
effective in curing disease, esp in surgically unsalvageable cases
Denosumab: Fully human monoclonal antibody that targets RANKL.
It has been shown objective changes in tumor composition, reduced bony
destruction and clinical benefit in patients with advanced/unresectable giant
cell tumor of bone. More effective than bisphosphonates in reducing skeletal
morbidity, and can delay metastasis.
Question 273
Figure 273 is the anteroposterior radiograph of the
distal tibia of a 12-year-old girl who sustained a recent
injury while playing basketball. She is tender over the
involved area. What is the most appropriate next step?
1.MR images
2.Open biopsy
3.Chest CT scans
4.Cast placement
5.Internal fixation
Question 273
Figure 273 is the anteroposterior radiograph of the
distal tibia of a 12-year-old girl who sustained a recent
injury while playing basketball. She is tender over the
involved area. What is the most appropriate next step?
1.MR images
2.Open biopsy
3.Chest CT scans
4.Cast placement
5.Internal fixation
Recommended Readings
•This patient as a non-ossfying fibroma (NOF)
•Most common benign bone tumor in kids, a benign
fibrogenic lesion
•Thought due to abnormal osteoclastic activity in
metaphysis
•Usually assymptomatic; have “bubbly” lytic lesions
surrounded by sclerotic bone
•If no fracture and assymptomatic, observe
•If frature, can treat as if just fracture alone.
• Ortiz EJ, Isler MH, Vania JE, Canosa R. Pathologic fractures in children. Clin Orthop Relat Res. 2005 Mar;(432):116-26.
• Temple HT, Scully SP, Aboulafia AJ. Benign bone tumors. Instr Course Lect. 2002;51:429-39.

You might also like